Тестове по клинична фармакология за студенти. Предизпитен тест по фармакология

Клинично фармакологичен тест

Система за подготовка на Gee Test oldkyx.com

Списък с въпроси по клинична фармакология

1. Полуживотът на лекарството е:
1) [-] времето за достигане на максималната концентрация на лекарството в плазмата;

2) [-] времето, през което лекарството достига системното кръвообращение;

3) [-] времето, през което лекарството се разпределя в тялото;

4) [+] времето, през което концентрацията на лекарството в плазмата намалява с 50%;

5) [-] времето, през което половината от приложената доза достига до прицелния орган.

2. Ширината на терапевтичното действие е:
1) [-] терапевтична доза от лекарството;

2) [-] съотношението на концентрацията на лекарството в орган или тъкан към концентрацията му в кръвната плазма;

3) [+] диапазонът между минималните терапевтични и минималните токсични концентрации на лекарството в плазмата;

4) [-] процент на несвързано с протеин лекарство;

5) [-] диапазон между минималната и максималната терапевтична концентрация на лекарството.

3. Конкурентните рецепторни лекарства включват:
1) [-] НСПВС (нестероидни противовъзпалителни средства);

2) [+] β-блокери;

3) [-] бримкови диуретици;

4) [-] нитрати;

5) [-] флуорохинолони.

4. Функцията на черния дроб и бъбреците трябва да се има предвид при предписване на следните лекарства:
1) [-] липофилен, образуващ неактивни метаболити;

2) [+] липофилен, образуващ активни метаболити;

3) [-] хидрофилен;

4) [-] хепатотоксичен;

5) [-] нефротоксичен.

5. Селективността на действието на лекарственото вещество зависи от:
1) [-] полуживот;

2) [-] метод на приемане;

3) [-] връзка с протеина;

4) [-] обем на разпределение;

5) [+] дози.

6. Кинетиката на насищане се характеризира с:
1) [+] увеличаване на полуживота на приложената доза с непроменен клирънс;

2) [-] скоростта на елиминиране е пропорционална на концентрацията на лекарството в плазмата и дозата;

3) [-] полуживотът не е пропорционален на приложената доза.

7. Фактор, определящ необходимостта от преизчисляване на режима на приложение на лекарството при хронична бъбречна недостатъчност:
1) [-] висока липофилност на лекарството;

2) [-] ниска връзка с плазмените протеини;

3) [-] наличието на активни тубулни екскреционни системи;

4) [+] висока степен на екскреция непроменена.

8. Какви лекарства преминават по-лесно през BBB?
1) [-] с висока разтворимост във вода;

2) [+] с висока разтворимост в мазнини;

3) [-] проявяващи свойствата на слаби киселини;

4) [-] проявяващи свойствата на слаби основи;

5) [-] със слаба връзка с плазмените протеини.

9. В кой случай е по-пълно усвояване?
1) [-] абсорбция от стомаха на лекарство, проявяващо свойствата на слаба основа;

2) [-] абсорбция от тънките черва на лекарство, проявяващо свойствата на слаба киселина;

3) [+] абсорбция от тънките черва на лекарство, което проявява свойствата на слаба основа.

10. Концепцията за "пресистемен метаболизъм" включва:
1) [+] биотрансформация на лекарства в черния дроб по време на първия пасаж и в червата;

2) [-] биотрансформация на лекарства в червата;

3) [-] биотрансформация на лекарства в черния дроб по време на първия пасаж и в бъбреците;

4) [-] биотрансформация на лекарства в черния дроб, бъбреците и червата.

11. Следната група странични ефекти е строго зависима от дозата:
1) [-] фармацевтични;

2) [-] фармакогенетичен;

3) [-] алергични;

4) [-] мутагенен;

5) [+] синдром на отнемане.

12. Определете група лекарства с тесен терапевтичен индекс:
1) [-] β-блокери;

2) [-] пеницилини;

3) [+] сърдечни гликозиди;

4) [-] АСЕ инхибитори;

5) [-] мощни диуретици.

13. Провеждането на лекарствен мониторинг е желателно при лечението на следната група лекарства:
1) [+] антиконвулсанти;

2) [-] β2-миметици на симптомите;

3) [-] пеницилини;

4) [-] глюкокортикоиди;

5) [-] М-холинолитици.

14. Забавените включват следната група странични ефекти:
1) [-] токсичен;

2) [-] развитие на лекарствена зависимост;

3) [-] фармакогенетичен;

4) [+] канцерогенен;

5) [-] синдром на отнемане.

15. Развитието на асистолия е възможно при комбинацията на пропранолол с:
1) [-] фенобарбитал;

2) [-] фуроземид;

3) [+] верапамил;

4) [-] фенитоин;

5) [-] ранитидин.

16. Рискът от токсични ефекти се увеличава, когато гентамицин се комбинира с:
1) [+] фуроземид;

2) [-] пеницилин;

3) [-] метилксантини;

4) [-] макролиди;

5) [-] глюкокортикоиди.

17. Рискът от нежелана бременност се увеличава, когато оралните контрацептиви се комбинират с:
1) [-] хипотония;

2) [-] витамин С;

3) [-] алкохол;

4) [+] тетрациклин;

5) [-] глюкокортикоиди.

18. При бъбречна патология се наблюдават следните промени във фармакокинетиката на лекарствата, с изключение на:
1) [-] нарушена бъбречна екскреция;

2) [-] повишаване на концентрацията на лекарства в кръвната плазма;

3) [-] намаляване на свързването с плазмените протеини;

4) [-] увеличение на T1 / 2;

5) [+] намаляване на бионаличността.

19. Цирозата на черния дроб се причинява от следните промени във фармакокинетиката на лекарствата, с изключение на:
1) [-] намаляване на метаболизма при първо преминаване;

3) [-] увеличение на T1 / 2;

4) [-] повишаване на бионаличността;

5) [+] намаляване на обема на разпределение.

20. При сърдечна недостатъчност се наблюдават следните промени във фармакокинетиката на дигоксин, с изключение на:
1) [-] намаляване на абсорбцията в стомашно-чревния тракт с 30%;

2) [-] намаляване на свързването с плазмените протеини;

3) [+] засилване на метаболизма в черния дроб;

4) [-] намаляване на бъбречната екскреция;

5) [-] увеличение на Т1/2.

21. Алкохолът в еднократна доза големи дози води до:
1) [-] повишена абсорбция на лекарството;

3) [+] забавяне на метаболизма в черния дроб;

4) [-] намаляване на бъбречната екскреция;

5) [-] увеличение на T1 / 2.

22. Никотинът води до:
1) [-] намаляване на абсорбцията на лекарството;

2) [-] увеличаване на обема на разпространение на лекарството;

3) [-] намаляване на връзката с плазмения протеин;

4) [+] повишен метаболизъм в черния дроб;

5) [-] повишена бъбречна екскреция на лекарства.

23. Обичайната форма на освобождаване на лекарството се характеризира с:
1) [-] нитронг;

2) [-] сутак-кърлеж;

3) [+] нитросорбид;

4) [-] нифедипин-GITS;

5) [-] верапамил SR.

24. За спиране на атака на ангина пекторис се използва сублингвална таблетна лекарствена форма:
1) [-] нитронг;

2) [-] сустак;

3) [+] нитросорбид;

4) [-] атенонолол;

5) [-] верапамил SR.

25. За да се предотврати развитието на толерантност при редовна употреба на нитрати, интервалът без нитрати трябва да бъде:
1) [-] 2-4 часа;

2) [-] 4-6 часа;

3) [-] 6-8 часа;

4) [+] 8-12 часа.

26. За засилване на антиангинозния ефект най-безопасната комбинация е:
1) [-] верапамил + пропранолол;

2) [-] верапамил + атенолол;

3) [-] верапамил + метопролол;

4) [+] верапамил + изосорбид динитрат;

5) [-] верапамил + дилтиазем.

27. Методите за оценка на антиангинозната ефикасност на лекарството са всички изброени по-долу, с изключение на:
1) [-] Холтер ЕКГ мониториране;

2) [+] мониториране на дневно кръвно налягане;

3) [-] стрес ехо;

4) [-] тест на бягаща пътека;

5) [-] VEM проби.

28. При болен от ангина пекторис в комбинация с артериална хипертония предимство имат следните лекарства:
1) [-] нитрати;

29. При ангина пекторис лекарствата на избор са лекарства от следния клас:

2) [+] блокери на β-адренергичните рецептори;

3) [-] α-адренергични блокери;

4) [-] имидазолинови рецепторни агонисти;

5) [-] ангиотензин II рецепторни блокери.

30. При вазоспастична стенокардия лекарствата на избор са лекарства от следния клас:
1) [-] блокери на хистаминови рецептори;

2) [-] блокери на β-адренергичните рецептори;

3) [-] α-адренергични блокери;

4) [+] блокери на калциевите канали;

5) [-] ангиотензин II рецепторни блокери.

31. Адекватен метод за проследяване на ефективността и безопасността на антихипертензивната терапия е:
1) [-] ежедневно ЕКГ мониториране;

2) [+] ежедневно проследяване на кръвното налягане;

3) [-] еднократни измервания на кръвното налягане;

4) [-] измерване на дихателната функция;

5) [-] динамика на QT интервала на ЕКГ.

32. Изберете нежелан ефект, който не е характерен за верапамил:
1) [-] брадикардия;

2) [-] запек;

3) [-] развитие на AV блокада;

4) [-] подуване на краката и стъпалата;

5) [+] бронхоспазъм.

33. За лечение на артериална хипертония лекарство на първи избор при пациент с хронична сърдечна недостатъчност е:
1) [+] еналаприл;

2) [-] верапамил;

3) [-] клонидин;

4) [-] празозин;

5) [-] нифедипин.

34. Посочете антихипертензивно лекарство, което повишава активността на симпатоадреналната система:
1) [+] нифедипин;

2) [-] клонидин;

3) [-] каптоприл;

4) [-] метопролол;

5) [-] ирбесартан.

35. При артериална хипертония в комбинация със синусова тахикардия се предпочитат:
1) [-] блокери на калциевите канали на дихидропиридинови производни;

2) [-] бримкови диуретици;

3) [+] β-блокери;

4) [-] α-блокери;

5) [-] тиазидни диуретици.

36. α1-блокерите са лекарства на избор за лечение на артериална хипертония:
1) [-] при пациенти с чернодробно заболяване;

2) [-] при пациенти с ритъмни нарушения;

3) [+] при възрастни мъже с аденом на простатата и затруднено уриниране;

4) [-] при пациенти с ангина пекторис;

5) [-] при пациенти с анамнеза за инфаркт на миокарда.

37. За лечение на артериална хипертония при пациенти с бронхиална астма не могат да се използват:
1) [-] блокери на калциевите канали;

2) [-] ангиотензин II рецепторни антагонисти;

3) [-] α1-блокери;

4) [+]β-блокери;

5) [-] диуретици.

38. Пациенти с артериална хипертония и анамнеза за инфаркт на миокарда трябва да бъдат предписани на първо място:
1) [+] β-блокери;

2) [-] диуретици;

3) [-] блокери на калциевите канали;

4) [-] имидазолинови рецепторни агонисти;

5) [-] α1-блокери.

39. Лекарства на първи избор при пациенти с артериална хипертония и тежка периферна артериална стеноза са:
1) [-] неселективни β-блокери;

2) [+] калциеви антагонисти;

3) [-] диуретици;

4) [-] ангиотензин II рецепторни блокери;

5) [-] α2-адренергични рецепторни агонисти.

40. Избройте групите лекарства, които подобряват прогнозата на пациенти с CHF:
1) [-] АСЕ инхибитори;

2) [-] β-блокери;

3) [-] ангиотензин II рецепторни блокери;

4) [-] спиронолактон;

41. Избройте лекарства, които имат директен положителен инотропен ефект:
1) [-] дигоксин;

2) [-] допамин;

3) [-] амринон;

4) [-] левосимендан;

5) [+] всички изброени лекарства.

42. Избройте β-блокери, които имат доказана ефикасност при лечението на пациенти със ЗСН:
1) [-] атенолол;

2) [-] пропранолол;

3) [+] карведилол;

4) [-] соталол;

43. Посочете показанията за назначаване на спиронолактон:
1) [-] нивото на калий в кръвта> 5,5 mmol/l;

2) [-] подуване на краката и стъпалата;

3) [+] сърдечна недостатъчност IV FC по класификацията на NYHA;

4) [-] креатининов клирънс под 30 ml/min;

44. Титрирането на дозата на АСЕ инхибиторите и β-блокерите при пациенти със ЗСН предполага:
1) [-] началото на терапията с минималната доза от лекарството;

2) [-] увеличаване на дозата на лекарството на всеки 2 седмици;

3) [-] постигане на целевата доза на лекарството;

4) [-] намаляване на броя на хоспитализациите и увеличаване на продължителността на живота на пациентите;

5) [+] всички изброени функции.

45. Показания за назначаване на амлодипин при CHF:
1) [-] застойна сърдечна недостатъчност;

2) [+] неконтролирани стойности на кръвното налягане;

3) [-] анамнеза за миокарден инфаркт;

4) [-] ритъмни нарушения;

5) [-] всички изброени функции.

46. ​​​​Принципи на диуретичната терапия при ХСН:
1) [-] назначаването на диуретици при сърдечна недостатъчност II-IV FC според класификацията на NYHA;

2) [-] загуба на тегло с 0,5-1,0 kg на ден;

3) [-] контрол на кръвното налягане;

4) [-] контрол на нивото на калий в кръвта;

5) [+] всичко по-горе.

47. Посочете предпочитания начин на приложение на лекарството при застойна сърдечна недостатъчност:
1) [-] сублингвално;

2) [-] ректално;

3) [+] парентерално;

4) [-] устно;

5) [-] всички изброени начини на приложение.

48. Ефекти на АСЕ инхибитор при пациенти със ЗСН:
1) [-] ефектът на АСЕ инхибитора върху смъртността зависи от продължителността на лечението;

2) [-] намаляването на риска от смърт е по-изразено при пациенти с по-висок FC;

3) [-] наличието на дозозависим ефект на АСЕ инхибитор при пациенти със сърдечна недостатъчност;

4) [+] всичко по-горе.

49. Избройте лекарствата, които увеличават продължителността на потенциала на действие:
1) [-] хинидин;

2) [-] прокаинамид;

3) [-] амиодарон;

4) [-] дигоксин;

5) [+] всички изброени лекарства.

50. Лекарства, които удължават QT интервала:
1) [-] клиндамицин;

2) [-] амиодарон;

3) [-] котримоксазол;

4) [-] хинидин;

5) [+] всички изброени лекарства.

51. Лекарства, които удължават PQ интервала:
1) [-] лидокаин;

2) [-] хинидин;

3) [-] дизопирамид;

4) [+] дигоксин;

5) [-] всички изброени лекарства за назначаване на антиаритмични лекарства.

52. Показания за предписване на антиаритмични средства са:
1) [-] чести ритъмни нарушения;

2) [-] камерни екстрасистоли - 6 в минута;

3) [-] нарушение на ритъма на високи градации;

4) [+] нарушение на хемодинамиката;

5) [-] всичко по-горе.

53. Избройте лекарства, които имат антихолинергични странични ефекти:
1) [-] амиодарон;

2) [-] верапамил;

3) [-] лидокаин;

4) [+] хинидин;

5) [-] всички изброени лекарства.

54. Избройте лекарства, които повишават прага на фибрилация:
1) [-] кордарон;

2) [-] бретилиев тозилат;

3) [-] соталол;

4) [-] пропранолол;

5) [+] всички изброени лекарства.

55. Избройте лекарствата, предписани за предотвратяване на аритмии при синдром на WPW:
1) [-] дигоксин;

2) [-] дилтиазем;

3) [+] амиодарон;

4) [-] прокаинамид;

5) [-] всички изброени лекарства.

56. Показания за възстановяване на ритъма с постоянна форма на предсърдно мъждене:
1) [-] чести епизоди на тахисистолия;

2) [-] слабост на синусовия възел;

3) [+] анамнеза за тромбоемболизъм;

4) [-] неефективност на провежданата лекарствена терапия;

5) [-] всичко по-горе.

57. Лекарство по избор за лечение на пароксизми на камерна тахикардия:
1) [-] лидокаин;

2) [-] прокаинамид;

3) [-] бретилий;

4) [-] пропафенон;

5) [+] всички изброени лекарства.

58. Странични ефекти на амиодарон:
1) [-] фоточувствителност;

2) [-] дисфункция на щитовидната жлеза;

3) [-] суха кашлица;

4) [-] преходно повишаване на активността на чернодробните аминотрансферази;

5) [+] всички изброени ефекти.

59. Показания за употреба на аденозин:
1) [-] пароксизъм на предсърдно мъждене;

2) [+] реципрочни суправентрикуларни тахикардии;

3) [-] камерна тахикардия;

4) [-] екстрасистол;

5) [-] всичко по-горе.

60. Метод за оценка на ефективността на антиаритмичната терапия в извънболничната практика:
1) [+] Холтер ЕКГ мониториране;

3) [-] тестове с физическа активност;

4) [-] EFI (електрофизиологично изследване);

5) [-] всички изброени методи.

61. Инхалаторните глюкокортикоиди включват:
1) [-] хидрокортизон;

2) [+] беклометазон;

3) [-] преднизолон;

4) [-] полкорталон;

5) [-] дексаметазон.

62. Дългодействащите селективни β2-агонисти включват:
1) [-] флутиказон;

2) [+] салметерол;

3) [-] салбутамол;

4) [-] фенотерол;

5) [-] тербуталин.

63. За спиране на пристъп на бронхиална астма се използва:
1) [-] тиотропиев бромид;

2) [-] теопек;

3) [-] натриев кромогликат;

4) [+] салбутамол;

5) [-] будезонид.

64. Дългодействащите антихолинергици включват:
1) [-] ипратропиев бромид;

2) [-] натриев кромгликат;

3) [+] тиотропиев бромид;

4) [-] окситропиев бромид;

5) [-] триамцинолон ацетонид.

65. Страничен ефект на инхалаторните глюкокортикостероиди е:
1) [-] главоболие;

2) [-] затлъстяване;

3) [+] орална кандидоза;

4) [-] захарен диабет;

5) [-] полиурия.

66. Муколитичните лекарства включват:
1) [-] кодеин;

2) [-] натриев кромогликат;

3) [+] ацетилцистеин;

4) [-] салметерол;

5) [-] теофилин.

67. При едновременна употреба се повишава концентрацията на теофилин в кръвта:
1) [+] офлоксацин;

2) [-] пеницилин;

3) [-] цефтриаксон;

4) [-] гентамицин;

5) [-] бисептол.

68. При едновременна употреба намалява концентрацията на теофилин в кръвта:
1) [-] пефлоксацин;

2) [-] циметидин;

3) [+] рифампицин;

4) [-] еритромицин;

5) [-] ампиокс.

69. Бронходилататорите не включват:
1) [-] метилксантини;

2) [-] антихолинергици;

3) [-] симпатикомиметици;

4) [+] блокери на левкотриенови рецептори.

70. Препаратът за базисна терапия при хроничен обструктивен бронхит е:
1) [+] тиотропиев бромид;

2) [-] недокромил натрий;

3) [-] фенотерол;

4) [-] монтелукаст;

5) [-] аминофилин.

71. Тахикардия като страничен ефект се развива при приемане на всички изброени лекарства, с изключение на:
1) [-] салбутамол;

2) [-] изопротеринол;

3) [-] фенотерол;

4) [-] теофилин;

5) [+] ипратропиев бромид.

72. Комбинираните лекарства за лечение на бронхиална астма не включват:
1) [-] дитек;

2) [-] серетид;

3) [-] симбикорт;

4) [+] беклазон;

5) [-] беродуал.

73. Инхалаторният глюкокортикостероид има най-изразен противовъзпалителен ефект:
1) [-] бекламетазон дипропионат;

2) [-] будезонид;

3) [-] триамцинолон ацетонид;

4) [+] флутиказон пропионат;

5) [-] флунизолид.

74. Лекарство на избор при наличие на инфекции, причинени от метицилин-резистентни щамове на Staphylococcus aureus е:
1) [-] азитромицин;

2) [-] метронидазол;

3) [-] гентамицин;

4) [+] линезолид;

5) [-] цефуроксим.

75. Изберете група антибактериални лекарства за лечение на инфекции, причинени от вътреклетъчни патогени:
1) [+] макролиди;

2) [-] пеницилини;

3) [-] аминогликозиди;

4) [-] цефалоспорини;

5) [-]сулфонамиди.

76. Посочете групата антибактериални лекарства с най-висока антианаеробна активност:
1) [-] гликопептиди;

2) [-] аминопеницилини;

3) [-] тетрациклини;

4) [-] аминогликозиди;

5) [+] нитроимидазоли.

77. Всички изброени антибактериални лекарства са нефротоксични, с изключение на:
1) [-] гентамицин;

2) [-] карбеницилин;

3) [+] азитромицин;

4) [-] цефазолин;

5) [-] ванкомицин.

78. Посочете антибактериално лекарство, неактивно срещу пневмококи:
1) [-] азитромицин;

2) [-] пеницилин;

3) [-] цефтриаксон;

4) [+] ципрофлоксацин;

5) [-] хлорамфеникол.

79. Изберете рационална комбинация от антибактериални лекарства, която има синергичен ефект срещу грам-положителни микроорганизми и безопасност:
1) [-] пеницилини + тетрациклини;

2) [-] пеницилини + цефалоспорини;

3) [-] аминогликозиди + гликопептиди;

4) [+] пеницилини + аминогликозиди;

5) [-] пеницилини + сулфонамиди.

80. Следните антибактериални лекарства проникват добре през кръвно-мозъчната бариера:
1) [-] линкозамиди;

2) [-] макролиди;

3) [-] тетрациклини;

4) [-] аминогликозиди;

5) [+] III поколение цефалоспорини.

81. Лекарството на избор при лобарна пневмония е:
1) [-] ципрофлоксацин;

2) [-] доксициклин;

3) [-] гентамицин;

4) [-] цефотаксим;

5) [+] бензилпеницилин.

82. Лекарство на избор при тонзилофарингит е:
1) [+] цефуроксим аксетил;

2) [-] доксициклин;

3) [-] цефтазидим;

4) [-] офлоксацин;

5) [-] фурагин.

83. Лекарства на избор при инфекции на жлъчните пътища са:
1) [-] аминогликозиди;

2) [-] нитрофурани;

3) [+] III поколение цефалоспорини;

4) [-] макролиди;

5) [-] естествени пеницилини.

84. При болнични инфекции, протичащи в интензивни отделения, лекарства на избор са следните комбинации от антибактериални лекарства:
1) [-] ампицилин + гентамицин;

2) [+] цефтазидим + амикацин;

3) [-] цефуроксим + еритромицин;

4) [-] клиндамицин + гентамицин;

5) [-] норфлоксацин + пеницилин.

85. Предпочитаната група антибактериални лекарства при лечение на хроничен простатит са:
1) [-] сулфонамиди;

2) [-] карбапенеми;

3) [-] хинолони;

4) [-] линкозамини;

5) [+] флуорохинолони.

86. Изберете лекарство, което максимално потиска секрецията на солна киселина:
1) [-] пирензепин;

2) [-] циметидин;

3) [-] мизопростол;

4) [-] антиациди;

5) [+] омепразол.

87. Максималният брой странични ефекти сред H2-блокерите има:
1) [+] циметидин;

2) [-] роксатидин;

3) [-] низатидин;

4) [-] ранитидин;

5) [-] фамотидин.

88. Инхибира цитохром P-450:
1) [-] омепразол;

2) [-] пирензепин;

3) [+] циметидин;

4) [-] фамотидин;

5) [-] лансопразол.

89. Синдромът на откат се причинява от:
1) [-] синтетични простагландини;

2) [-] антиациди;

4) [-] М-холинолитици;

5) [+] H2 блокери.

90. При язва на дванадесетопръстника е рационално да се предписват антиациди:
1) [-] преди хранене;

2) [-] по време на хранене;

3) [+] 1,5-2 часа след хранене;

4) [-] 5 часа след хранене;

5) [-] независимо от приема на храна.

91. За профилактика на язви, причинени от НСПВС, следните са най-ефективни:
1) [-] антиациди;

2) [-] H2 блокери;

3) [-] блокери на "протонна помпа";

4) [+] синтетични простагландини;

5) [-] М-холинолитици.

92. При ерадикационна терапия за унищожаване на H. pylori се използва следният антибиотик:
1) [-] карбеницилин;

2) [-] еритромицин;

3) [-] цефоперазон;

4) [+] кларитромицин;

5) [-] хлорамфеникол.

93. От голямо практическо значение е развитието на резистентност на H. pylori към:
1) [-] ванкомицин;

2) [+] метронидазол;

3) [-] тетрациклин;

4) [-] нитрофурани;

5) [-] цефотаксим.

94. Бактерицидно действие срещу H. pylori има:
1) [-] сукралфат (вентер);

2) [+] бисмутов субцитрат (де-нол);

3) [-] алмагел;

4) [-] фамотидин;

5) [-] пирензепин.

95. При ерадикационната терапия за унищожаване на H. pylori се използват:
1) [-] антиациди;

2) [-] синтетични простагландини;

3) [+] блокери на "протонната помпа";

4) [-] М-холинолитици;

5) [-] гликопептиди.

96. Кортикостероидите повишават токсичността:
1) [-] теофилин;

2) [+] тиазидни диуретици;

3) [-] препарати от злато;

4) [-] противоязвени лекарства.

97. Ефектът на глюкокортикоидите намалява:
1) [-] циметидин;

2) [-] аспирин;

3) [-] диклофенак;

4) [-] амиодарон;

5) [+] Рифампицин.

98. Най-висока минералкортикоидна активност има:
1) [-] полкартолон;

2) [-] преднизолон;

3) [+] хидрокортизон;

4) [-] дексаметазон.

99. След прием на НСПВС ефектът се развива по-бързо:
1) [-] противовъзпалително;

2) [+] аналгетик;

3) [-] антикоагулант.

100. Най-силно изразено аналгетично свойство има:
1) [-] ацетилсалицилова киселина;

2) [-] ибупрофен;

3) [-] напроксен;

4) [+] парацетамол.

101. Механизмът на развитие на улцерогенния ефект на НСПВС е:
1) [-] повишена киселинност на стомашния сок;

2) [+] намаляване на синтеза на простагландини в стомашната лигавица;

3) [-] намалено възстановяване на лигавицата.

102. Най-силно изразено противовъзпалително свойство има:
1) [+] фенилбутазон;

2) [-] метамизол;

3) [-] пироксикам;

4) [-] парацетамол;

5) [-] ибупрофен.

103. Ранен страничен ефект на глюкокортикоидите е:
1) [-] катаракта;

2) [-] миопатия;

3) [-] остеопороза;

4) [-] Кушингоиден синдром;

5) [+] стероиден диабет.

104. Посочете ефекта, който не е характерен за кортикостероидите:
1) [-] противовъзпалително;

2) [-] антиалергично;

3) [-] антишок;

4) [-] имуносупресивен;

Преглед:

Предмет: "PMRS, засягащ периферната нервна система"

Тестови задачи

1. Адреналинът причинява:

Изберете един отговор.

а.) Намалена консумация на кислород

Б.) Хипергликемия

В.) Инхибиране на гликогенолизата

D.) Инхибиране на липолизата

2. Адреналинът е противопоказан при:

Изберете един отговор.

а.) Тиреотоксикоза

Б.) Анафилактичен шок

В.) Сърдечен блок

D.) Хипогликемична кома

3. Ганглиоблокатор:

Изберете един отговор.

а.) атропин;

B.) пипекуроний;

В.) пентамин;

D.) сукцинилхолин (дитилин).

4. Ганглиоблокерите се използват за лечение на:

Изберете един отговор.

а.) запек.

Б.) хипертонична криза;

В.) глаукома;

Г.) задържане на урина;

5. Действието на M-XR агониста е блокирано:

Изберете един отговор.

а.) Цитизин

Б.) Тубокурарин

В.) Прозерин

D.) Атропин

Д.) Пилокарпин

6. За да спрете действието на конкурентните мускулни релаксанти, приложете:

Изберете един отговор.

а.) атропин;

Б.) дипироксим.

В.) неостигмин (прозерин);

7. Селективен М-холиномиметик (агонист на мускариновите холинергични рецептори):

Изберете един отговор.

а.) Прозерин

Б.) Пилокарпин

В.) Цитизин

D.) Физостигмин

E.) Карбахолин

8. Адсорбентите включват:

Изберете един отговор.

а.) Слуз от нишесте.

Б.) Отвара от дъбови кори;

В.) Танин;

Г.) Активен въглен;

9. Всички изброени по-долу са дразнители с изключение на:

Изберете един отговор.

а.) Основен бисмутов нитрат;

Б.) Ментол.

В.) Пречистено терпентиново масло (терпентин);

D.) Хартия с горчица;

10.M-антихолинергични:

Изберете един отговор.

а.) пентамин;

B.) пипекуроний;

В.) сукцинилхолин (дитилин).

D.) атропин;

11.M-антихолинергиците причиняват развитието на мидриаза:

Изберете един отговор.

а.) повишаване на тонуса на радиалния мускул на ириса;

Б.) намаляване на тонуса на циркулярния мускул на ириса;

В.) повишаване на тонуса на цилиарния мускул.

12.M-антихолинергиците се използват за лечение на:

Изберете един отговор.

а.) артериална хипертония;

Б.) глаукома;

В.) стомашна язва.

D.) миастения гравис;

13.М-антихолинергиците са противопоказани при:

Изберете един отговор.

а.) бронхиална астма;

Б.) глаукома;

В.) атриовентрикуларна блокада;

D.) стомашна язва.

14.М-холиномиметиците, за разлика от инхибиторите на ChE, не засягат холинергичната синаптична трансмисия:

Изберете един отговор.

а.) в нервно-мускулния синапс

B.) от постганглионарните аксони на автономните нерви към ефектора (гладки мускули, екзокринни жлези)

В.) в ЦНС

15. Местните анестетици се използват в комбинация с адреналин, защото:

Изберете един отговор.

а.) абсорбцията на анестетика се ускорява и локалният анестетичен ефект се засилва.

Б.) абсорбцията на анестетика се забавя и се засилва локалноанестетичният ефект;

В.) абсорбцията на анестетика се забавя и местният анестетичен ефект отслабва;

16. Метопролол е показан за лечение на:

Изберете един отговор.

а.) Атриовентрикуларен блок

Б.) Бронхиална астма

В.) Доброкачествена хиперплазия на простатата

Г.) Артериална хипертония

17. Механизмът на действие на адстрингентите се дължи на:

Изберете един отговор.

а.) блок на натриев канал;

Б.) адсорбция на химични съединения;

В.) покриване на лигавиците с филм, който предотвратява дразненето на сетивните нерви. г.) ​​коагулация на протеини и образуване на филм, който предпазва окончанията на сетивните нерви от дразнене;

18. Механизмът на действие на локалните анестетици се дължи на:

Изберете един отговор.

а.) Блокиране на калциевите канали и удължаване на абсолютния рефрактерен период;

Б.) Блокиране на калиеви канали и невъзможност за реполяризация на мембраната;

В.) Активиране на хлоридни канали и хиперполяризация.

Г.) Блок на натриевите канали и невъзможност за деполяризация на мембраната;

19. Мускулен релаксант:

Изберете един отговор.

а.) скополамин.

B.) пипекуроний;

В.) атропин;

D.) пентамин;

20. Нежелан страничен ефект на ганглийните блокери:

Изберете един отговор.

а.) хипертонична криза;

Б.) повишено вътреочно налягане.

В.) ортостатичен колапс;

Г.) бронхоспазъм;

21. Неселективен бета-блокер:

Изберете един отговор.

а.) Метопролол

Б.) Атенолол

В.) Празозин

D.) Пропранолол

22. Неостигмин (прозерин) се използва за лечение на миастения гравис, тъй като подобрява холинергичното синаптично предаване:

Изберете един отговор.

а.) В автономния ганглий

B.) На мионевралната връзка

C.) От постганглионарни холинергични влакна до клетки на ефекторни органи

23. Норепинефринът се повишава:

Изберете един отговор.

а.) Периферно съдово съпротивление

Б.) Мотилитет на стомашно-чревния тракт

В.) Бронхиален тонус

D.) Сърдечна честота

24. Защо атропинът (третичен амин) превъзхожда метацина (кватернерно амониево съединение) по отношение на действието на ЦНС:

Изберете един отговор.

а.) по-добре разпределени в тялото (> Vd стойности);

B.) по-добре се абсорбира в системното кръвообращение от мястото на инжектиране (> коефициент на бионаличност);

C.) се екскретира (елиминира) по-бавно от тялото (> стойности на T1 / 2).

25. Защо галантаминът (третичен амин) превъзхожда прозерина (кватернерно амониево съединение) по отношение на действието на ЦНС:

Изберете един отговор.

а.) По-бавно се екскретира (елиминира) от тялото (> T1 / 2 стойности)

B.) По-добре се абсорбира в системното кръвообращение от мястото на инжектиране (> фактор на бионаличност)

C.) По-добро разпределение в тялото (> Vd стойности)

26. Празозинът причинява:

Изберете един отговор.

а.) Намален тонус на гладката мускулатура на бронхите

Б.) Намалено вътреочно налягане

В.) Намаляване и отслабване на сърдечните контракции

D.) Намалено периферно съдово съпротивление

27. Пропранололът причинява:

Изберете един отговор.

а.) Намален стомашно-чревен мотилитет

B.) Контракция на мускула на ириса (мидриаза)

В.) Намален бронхиален тонус

D.) Намален пулс

28. Рефлекторната брадикардия причинява:

Изберете един отговор.

а.) Салбутамол

B.) Празозин

В.) Метопролол

D.) Карведилол

Д.) Норепинефрин

29. Съкратителната активност на миометриума се намалява от:

Изберете един отговор.

а.) Салбутамол

Б.) Карведилол

В.) Пропранолол

D.) Норепинефрин

Д.) Метопролол

30. Средства за лечение на остра съдова недостатъчност:

Изберете един отговор.

а.) Метопролол

Б.) Салбутамол

В.) Добутамин

D.) Норепинефрин

E.) Пропранолол

31. Само за повърхностна анестезия се използва:

Изберете един отговор.

а.) Бензокаин (анестезин).

B.) Бупивакаин;

В.) Прокаин (новокаин);

D.) Лидокаин;

32. Холиномиметиците са противопоказани при:

Изберете един отговор.

а.) Миастения гравис

Б.) Болест на Алцхаймер

В.) Бронхиална астма

D.) Ксеростомия

Д.) Глаукома

Преглед:

Предмет: "Химиотерапевтични средства"

Тестови задачи

1. Механизмът на действие на AG върху протеиновия синтез в микробните клетки се основава на способността им да инхибират:

Изберете един отговор.

а.) ДНК полимераза

Б.) РНК полимераза

В.) процес на транспептидация

D.) Процес на четене на иРНК код

2. Изберете антибиотик - инхибитор на протеиновия синтез в бактериална клетка:

Изберете един отговор.

а.) бензилпеницилин

Б.) карбеницилин

В.) ампицилин

D.) гентамицин

3. Изберете определение за "химиотерапия"

Изберете един отговор.

а.) химиотерапията е потискане на патогени на повърхността на човешкото тяло (кожа, лигавици)

Б) Химиотерапията е потискане на патогени в околната среда (предмети за грижа, инструменти, изписвания на пациенти)

В.) химиотерапията е въздействие върху клетките на макроорганизма

Г.) химиотерапията е потискане на патогени във вътрешната среда на макроорганизма

4. Принципите на химиотерапията включват следното:

Изберете един отговор.

а.) всички отговори са верни

Б.) Лечението с антибактериални лекарства трябва да започне възможно най-скоро след началото на заболяването

В.) лекарството трябва да бъде избрано, като се вземе предвид чувствителността на патогена към химиотерапевтичния агент;

D.) дозата на химиотерапевтичния агент трябва да се предписва, като се вземе предвид тежестта на заболяването при пациента

5. Нитрофурановите производни включват:

Изберете един отговор.

а.) Фталилсулфатиазол (фталазол)

Б.) налидиксова киселина

В.) фуразолидон

D.) нитрохексолин

6. Кои от следните антибиотици нарушават синтеза на клетъчната стена:

Изберете един отговор.

а.) хлорамфеникол

Б.) тетрациклини

В.) бета-лактамни антибиотици

D.) полимиксини

7. Какви лекарства за лечение на онихомикоза дават най-ниска честота на рецидив?

Изберете един отговор.

а.) тербинафин и итраконазол;

Б.) амфотерицин В и нистатин;

В.) гризеофулвин и леворин;

D.) цинков ундециленат и йод

8. Какви лекарства са ефективни срещу респираторни синцитиални вируси и грипни вируси?

Изберете един отговор.

а.) рибавирин, интерферон;

Б.) озелтамивир, ремантадин

C.) азидотимидин, саквинавир;

D.) ацикловир, фамцикловир;

9. Кое от твърденията правилно отразява един от общите принципи на химиотерапията при инфекциозно заболяване.

Изберете един отговор.

B.) Клиничното подобрение е основание за прекратяване на терапията

В.) ефективността на лечението често не зависи от продължителността на антибиотичната терапия. д. 3) след клинично подобрение лечението не трябва да се спира и да продължи, ако е необходимо, още 48-72 часа.

10. Какъв антибиотик е ефективен при псевдомембранозен колит?

Изберете един отговор.

а.) диклоксациклин

Б.) фуразолидон

В.) ванкомицин

D.) ампицилин

11. Кой от следните антибиотици принадлежи към бета-лактамните антибиотици:

Изберете един отговор.

а.) стрептомицин

Б.) меропенем

В.) тетрациклин

D.) полимиксин

12. Кое от химиотерапевтичните лекарства принадлежи към сулфонамидите:

Изберете един отговор.

а.) менкомицин

Б.) еритромицин

В.) стрептомицин

D.) сулфадимидин

13. Кой макролид има най-нисък клирънс?

Изберете един отговор.

а.) еритромицин

B.) азитромицин

В.) кларитромицин

D.) рокситромицин

14. Кое перорално лекарство е ефективно при гъбичен менингит (напр. криптококов)?

Изберете един отговор.

а.) амфотерицин В;

Б.) флуконазол

C.) тербинафин;

D.) кетоконазол;

15. Какъв препарат на бензилпеницилин принадлежи към биосинтетика:

Изберете един отговор.

а) ампицилин

B.) бензилпеницилин-бензатин

В.) азлоцилин

D.) карбеницилин

16. Какво лекарство се използва за лечение на гастроинтестинална кандидоза?

Изберете един отговор.

а) клотримазол

Б.) гризеофулвин;

В.) нитрофунгин;

D.) нистатин;

17. Какво лекарство се използва за лечение на системни микози?

Изберете един отговор.

а.) нистатин;

Б.) клотримазол

В.) амфотерицин В;

D.) гризеофулвин;

18. Кое лекарство е ефективно срещу вирусите на грип А и В?

Изберете един отговор.

а.) ремантадин

Б.) ацикловир;

C.) азидотимидин;

D.) озелтамивир;

19. Механизмът на действие на сулфонамидите е свързан с:

Изберете един отговор.

а) инхибиране на COX

B.) инхибиране на дихидрофолат редуктазата;

C.) конкурентен антагонизъм с PABA и инхибиране на дихидроптероат синтетазата

D.) конкурентен антагонизъм с GABA

20. Най-честото усложнение на бета-лактамните антибиотици е

Изберете един отговор.

а.) аритмии

B.) потискане на хематопоезата

В. 1) алергични реакции

D.) загуба на слуха

21. Полимиксините се считат за лекарства от 3-та линия („дълбок резерв“), защото:

Изберете един отговор.

а.) имат ниска ефективност

Б.) широко разпространена съпротива срещу тях

В.) поради ниска антимикробна активност

D.) поради висока органотоксичност

22. При използване на резорбтивни сулфонамиди са възможни следните нежелани реакции:

Изберете един отговор.

а.) агранулоцитоза

Б.) всичко по-горе

В.) кристалурия

Г.) хемолитична анемия, метхемоглобинемия

23. Антивирусните агенти (PVA) са най-ефективни, когато се лекуват рано, защото:

Изберете един отговор.

а.) PVAs показват визуален ефект;

B.) PVAs проявяват вируциден ефект;

C.) PVAs не показват органотоксичност

D.) PVA са органотоксични;

24. Посочете антиретровирусни лекарства (за лечение на HIV инфекция):

Изберете един отговор.

а.) арбидол, озелтамивир;

B.) азидотимидин, саквинавир;

В.) ацикловир, фамцикловир;

D.) интерферон, ганцикловир

25. Посочете механизма на действие на флуорохинолоните:

Изберете един отговор.

а.) увеличаване на пропускливостта на CPM

B.) Инхибиране на синтеза на бактериална стена

C.) инхибиране на PDEase

D.) инхибиране на ДНК гираза

26. Посочете лекарството, свързано с оксазолидиноните:

Изберете един отговор.

а.) линезолид

B.) моксифлоксацин

В.) ко-тримоксазол

D.) линкомицин

27. Посочете антихерпетично средство:

Изберете един отговор.

а.) азидотимидин;

Б.) ацикловир;

В.) арбидол;

D.) саквинавир

28. Какво е характерно за доксициклин?

Изберете един отговор.

а.) лошо се абсорбира от стомашно-чревния тракт

Б.) намалена бионаличност, когато се приема с храна

C.) T1/2 16-24 часа

D.) основният път на екскреция през MVP

Преглед:

Предмет : "Обща фармакология"

Тестови задачи

1 . Веществата с афинитет и вътрешна активност се наричат:

Изберете един отговор.

а.) антагонисти

B.) агонисти

2 . Действието на веществата, които се развиват след навлизането му в системното кръвообращение, се нарича:

Изберете един отговор.

а.) резорбтивна

Б.) местен

C.) страна

Г.) рефлекс

3 . Как се нарича действието на веществото, ако то взаимодейства само с функционално недвусмислени рецептори в определена локализация и не засяга други рецептори?

Изберете един отговор.

а) рефлекс

Б.) обратимо

В.) необратимо

Г.) селективен

4 . Какво е името на натрупването на лекарства в тялото, с многократното му приложение?

Изберете един отговор.

а.) тахифилаксия

Б.) материална кумулация

В.) идиосинкразия

D.) сенсибилизация

5 . Как се нарича намаляването на ефективността на действието на веществото с многократното му приложение?

Изберете един отговор.

а.) толерантност (пристрастяване)

Б.) кумулация

В.) идиосинкразия

Г.) пристрастяване

6. Какво е името на феномена, когато отнемането на наркотици причинява психични и соматични разстройства, свързани с дисфункции на много системи на тялото до смърт?

Изберете един отговор.

а.) синдром на отнемане

Б.) въздържание

В.) сенсибилизация

Г.) идиосинкразия

7. Кой от процесите протича във фазата на биотрансформация, която се нарича конюгация?

Изберете един отговор.

а.) хидролиза

Б.) възстановяване

В.) подкисляване

D.) ацетилиране

8. Кой отговор е най-съвместим с термина "рецептор"?

Изберете един отговор.

а.) активни групи от макромолекули на субстрати, с които лекарственото вещество взаимодейства

B.) транспортни системи, активирани от лекарства

C.) активирани от лекарства редокс ензими

Г.) йонни канали на биологични мембрани, чиято пропускливост се променя от лекарственото вещество

9. Какъв параметър на фармакокинетиката е обозначен като "T1 / 2":

Изберете един отговор.

а.) константа на скоростта на елиминиране

Б.) период на полуразпад (полуживот, период на полуразпад) на веществата

В.) абсорбция от мястото на инжектиране на 50% от веществото

D.) общ просвет

10. Метаболитната биотрансформация е:

Изберете един отговор.

а.) взаимодействие с глюкуронова киселина

Б.) превръщането на вещество в резултат на окисление, редукция, хидролиза

В.) свързване с плазмения албумин

Г.) метилиране и ацетилиране на вещества

11. Обемът на разпространение на лекарството отразява:

Изберете един отговор.

а.) съотношението на еднократната и дневната доза на лекарственото вещество

B.) Хипотетичен обем течност, в която е разпределено лекарството

C.) Очаквано количество лекарство, достигащо системното кръвообращение

D.) съотношение доза-тегло

12. Обемът на разпределение е нисък, ако:

Изберете един отговор.

а.) веществото е в плазмата, в интерстициалната и вътреклетъчната течност и се натрупва в тъканите

B.) веществото е в плазмата и в интерстициалната течност

В.) веществото е в плазмата, в интерстициалната и вътреклетъчната течност

D.) веществото се натрупва в кръвната плазма

13. Обърнете внимание на основния механизъм на усвояване на лекарствените вещества:

Изберете един отговор.

а.) пиноцитоза

Б.) пасивна дифузия

В.) активен транспорт

D.) филтриране

14. Фармакокинетиката включва:

Изберете един отговор.

а.) биотрансформация на лекарства в организма

Б.) Ефектът на лекарствата върху генетичния апарат

В.) усложнения на лекарствената терапия

Г.) Ефект на лекарствата върху метаболизма в организма

15. Какво включва понятието фармакодинамика?

Изберете един отговор.

а.) метаболизъм на лекарствата в организма

Б.) условия на съхранение на лекарствата

В.) Биологични ефекти на лекарствата

D.) метод на приложение на лекарството

16. Какво включва понятието "Биотрансформация":

Изберете един отговор.

а.) свързване на вещества с плазмените протеини

Б.) натрупване на вещества в мастната тъкан

В.) комплекс от физикохимични и биохимични трансформации на лекарствено вещество, насочени към отстраняването му от тялото

D.) натрупване на лекарство в мускулната тъкан

17. Какво се нарича вътрешна активност на веществото?

Изберете един отговор.

а.) способността на веществото да го разпознае, когато взаимодейства с рецептор

Б.) способността на веществото да взаимодейства с транспортни системи

В.) способността на веществото, когато взаимодейства с рецептор, да го стимулира и да предизвика биологичен ефект

D.) способността на веществото да взаимодейства с плазмените протеини

18. Какво означава терминът "афинитет"?

Изберете един отговор.

а.) афинитетът на дадено вещество към транспортните системи на тялото

Б.) афинитетът на веществото към албумините в кръвната плазма

В.) афинитет на лекарства към микрозомални чернодробни ензими

Г.) афинитетът на вещество към рецептор, водещ до образуването на комплекс „вещество-рецептор“ с него

19. Какво означава терминът "бионаличност":

Изберете един отговор.

а.) степен на свързване на веществата с плазмените протеини

Б.) количеството на веществото в урината спрямо началната доза на лекарството

В.) способността за преминаване през кръвно-мозъчната бариера

D.) количеството непроменено вещество, достигнало кръвната плазма, спрямо началната доза на лекарството

20. Какво съответства на понятието "активен транспорт":

Изберете един отговор.

а.) инвагинация на клетъчната мембрана с образуване на вакуола

B.) транспорт срещу градиент на концентрация с разход на енергия

C.) транспорт по концентрационен градиент без консумация на енергия

D. 1) улеснена дифузия

Преглед:

Предмет : "Имунотропни средства"

Тестови задачи

1. Блокерите на Н1-хистаминовите рецептори се използват за всички изброени по-долу показания ОСВЕН:

Изберете един отговор.

а.) уртикария;

Б.) бронхиална астма

В.) лекарствена алергия;

Г.) сезонен ринит;

2. Какви видове фармакодинамични ефекти на глюкокортикоидите се използват в медицината?

Изберете един отговор.

а.) всички са верни с изключение на 1

Б.) хипергликемия, потискане на растежните зони на епифизите;

В.) противошокови, детоксикиращи (индукция на чернодробни ензими);

Г.) всичко по-горе;

Д.) имуносупресивно, антиалергично, противовъзпалително;

3. Кои лекарства са най-ефективни като противовъзпалителни средства?

Изберете един отговор.

а.) широкоспектърни антибиотици

B.) НСПВС;

В.) SPVS;

D.) стабилизатори на мембраната на мастоцитите;

4. Какви са приемливите критерии за ефективност на инсулиновата терапия?

Изберете един отговор.

а.) еугликемия, еуглюкозурия;

Б.) еугликемия, аглюкозурия;

В.) агликемия, аглюкозурия

Г.) нормогликемия, еуглюкозурия;

5. Какви са локалните нежелани реакции от системното използване на GCS-съдържащи мехлеми и кремове?

Изберете един отговор.

а.) подуване, хиперемия, болезненост;

Б.) остеопороза, хирзутизъм, дисменорея

В.) хипертрофия, хиперпигментация, кандидоза;

Г.) повишен риск от локални инфекции, атрофия, депигментация;

6. Кои са най-опасните странични ефекти на системните кортикостероиди при продължителна употреба?

Изберете един отговор.

а) синдром на отнемане (надбъбречна недостатъчност);

B.) всичко по-горе;

В.) Синдром на Иценко-Кушинг ("Кушингоид");

Г.) 1 и 2 са верни.

Д.) имунодефицитно състояние;

7. Какви са показанията за употребата на гестагени?

Изберете един отговор.

а.) хормонозаместителна терапия след овариохистеректомия;

Б.) рак на гърдата, рак на простатата;

В.) дисфункционално маточно кървене, повтарящ се спонтанен аборт, ендометриоза, контрацепция;

D.) контрацепция при жени с висока хипердислипидемия, захарен диабет, анамнеза за холестаза

8. Кое е най-честото усложнение при лечение с инсулинови препарати?

Изберете един отговор.

а.) липодистрофия;

Б.) хипокалиемия;

В.) инсулинова резистентност

Г.) хипогликемия;

9. Кое GCS лекарство има ниска бионаличност при локално приложение (върху кожата)?

Изберете един отговор.

а.) будезонид;

Б.) флуоцинолон ацетонид (синафлан);

В.) преднизолон хемисукцинат

D.) бекламетазон пропионат;

10. Кое инхалаторно кортикостероидно лекарство има най-нисък риск от системни нежелани реакции?

Изберете един отговор.

а.) преднизолон хемисукцинат

Б.) бекламетазон пропионат;

C.) будезонид;

D.) флуоцинолон ацетонид (синафлан);

11. Кое лекарство принадлежи към инсулиновите сенсибилизатори?

Изберете един отговор.

а.) метформин;

Б.) хумулин

C.) пиоглитазон;

D.) акарбоза;

E.) глибенкламид;

12. Какво лекарство се използва при сезонни алергични реакции (сенна хрема) само като превантивно средство?

Изберете един отговор.

а.) клемастин;

Б.) хидрокортизон;

C.) натриев кромогликат;

Г.) всичко по-горе

13.Н1-хистаминовите блокери от второ поколение се различават от лекарствата от първото поколение

Изберете един отговор.

а.) изразен седативен ефект;

Б.) антиеметично действие

В.) значително М-антихолинергично действие;

Г.) по-голяма селективност на действие;

14. Окситоцинът се характеризира с всички свойства ОСВЕН

Изберете един отговор.

а.) чувствителността на матката е постоянно висока

Б.) ефективен в малки дози като стимулант;

В.) ефективен във високи дози като утеротоник;

Г.) чувствителността на матката към него се повишава за раждане;

15. Показанията за употреба на цитостатични имуносупресори включват всички изброени по-долу ОСВЕН:

Изберете един отговор.

а.) автоимунни заболявания;

B.) Предотвратяване на гниене

В.) тежки анафилактични реакции;

16. При хипотиреоидизъм се използва като средство за заместителна терапия

Изберете един отговор.

а.) протирелин;

Б.) калиев йодид;

В.) тиреотропин

D.) левотироксин;

17. Използването на цитостатични имуносупресори (метотрексат, флуороурацил, циклофосфамид) често е сложно

Изберете един отговор.

а.) левкопения и инфекциозен синдром;

Б.) алергии и фотодерматити;

В.) кървене и анемия;

D.) сънливост и летаргия

18. Тиамазол (мерказолил) като средство за основно (продължително) лечение е показан при ...

Изберете един отговор.

а.) рак на щитовидната жлеза;

Б.) микседем

В.) нодуларна токсична гуша;

Г.) дифузна токсична гуша;

19. Посочете правилната последователност на прилагане на лекарства за анафилактичен шок:

Изберете един отговор.

а.) преднизолон - клемастин - аминофилин - епинефрин;

Б.) клемастин (тавегил) - епинефрин (адреналин) - преднизолон - аминофилин (еуфилин)

в.) епинефрин - преднизолон - клемастин - аминофилин

20. Как да спрем диабетна кома?

Изберете един отговор.

а.) IV 40-80 ml 40% разтвор на глюкоза;

B.) IV 1 ml 0,1% разтвор на адреналин

C.) IV 20 IU инсулин-цинкова суспензия;

D.) интравенозно при 0,1 U/час краткодействащ инсулин;

21. Какво НЕ е абсолютно противопоказание за употребата на естрогенни препарати?

Изберете един отговор.

а.) маточно кървене с неизвестен характер;

Б.) чернодробно заболяване, анамнеза за жълтеница;

Д.) тромбофилия;

Преглед:

Предмет : „Лекарства, засягащи

на функцията на изпълнителните органи"

Тестови задачи

1. Лекарство за спешно лечение на хипертонична криза (с проява или увеличаване на признаци на увреждане на "целевите органи"):

Изберете един отговор.

а.) метилдопа;

Б.) каптоприл;

В.) натриев нитропрусид

D.) метопролол;

2.ААС за лечение на суправентрикуларни и камерни тахиаритмии:

Изберете един отговор.

а.) Верапамил

Б.) Лидокаин

C.) Прокаинамид (новокаинамид)

D.) Фенитоин (дифенин)

3.ААС с минимален потенциал за проаритмично действие:

Изберете един отговор.

а.) Пропранолол (индерал)

Б.) Амиодарон

В.) Пропафенон

D.) Лидокаин

4. AAS, използвани за лечение на коронарна артериална болест:

Изберете един отговор.

а.) Лидокаин

Б.) Верапамил

В.) Хинидин

D.) Пропафенон

5.AAS, характеризиращ се с най-дълъг полуживот:

Изберете един отговор.

а.) Хинидин

Б.) Аденозин

В.) Лидокаин

D.) Амиодарон

6. Антихипертензивен невротропен агент с периферно действие:

Изберете един отговор.

а.) каптоприл;

Б.) метопролол;

В.) нифедипин

D.) клонидин;

7. Антихипертензивно средство от групата на блокерите на калциевите канали:

Изберете един отговор.

а.) нифедипин

Б.) метопролол;

В.) каптоприл;

D.) лозартан;

8. Антихипертензивно средство от групата на миотропните вазодилататори:

Изберете един отговор.

а.) каптоприл;

Б.) дилтиазем;

C.) дихлотиазид;

D.) метопролол

9. Антихипертензивен агент с централно действие:

Изберете един отговор.

а.) клонидин;

Б.) пентамин

В.) натриев нитропрусид;

D.) каптоприл;

10. Антихипертензивно средство, което блокира алфа и бета адренорецепторите:.

Изберете един отговор.

а.) карведилол;

Б.) метопролол

C.) атенолол;

D.) пропранолол;

11. Антихипертензивно лекарство с висок риск от развитие на ефекта от първата доза (тежка хипотония в ортостатично положение):

Изберете един отговор.

а.) метопролол;

B.) хидрохлоротиазид;

В.) каптоприл;

D.) празозин

12. Антихипертензивно лекарство, противопоказано при двустранна стеноза на бъбречната артерия:

Изберете един отговор.

а.) метопролол;

Б.) верапамил;

В.) каптоприл;

D.) нифедипин

13. Антихипертензивно средство, което намалява образуването на ангиотензин II:

Изберете един отговор.

а.) верапамил;

B.) лосартан;

В.) каптоприл;

D.) празозин

14. Антихипертензивно средство, което намалява нивото на ренин в кръвта:

Изберете един отговор.

а.) празозин;

Б.) верапамил;

В.) пропранолол

D.) пентамин;

15. Антифибринолитично действие има:

Изберете един отговор.

а.) фитоменадион

Б.) калциев хлорид

В.) хепарин

D.) аминокапронова киселина

16. Бета-блокерите се използват за ИБС въз основа на:

Изберете един отговор.

а.) намаляване на нуждата от кислород чрез намаляване на сърдечната честота и контрактилитета на миокарда; b.) намаляване на нуждата от кислород на миокарда чрез намаляване на предварителното натоварване;

В.) увеличаване на извличането на O2 от кръвта

Г.) подобряване на коронарния кръвоток;

17. Вещество, което активира фибринолизата е:

Изберете един отговор.

а.) Варфарин

Б.) клопидогрел

В.) хирудин

D.) стрептокиназа

18. Всички кардиотоници повишават:

Изберете един отговор.

а.) атриовентрикуларна проводимост;

Б.) контрактилитет на миокарда;

В.) автоматизм на синоатриалния възел

D.) потребност от кислород на миокарда;

19. Основният елемент на антихипертензивното действие на алфа-блокерите:

Изберете един отговор.

а.) Венозна вазодилатация;

B.) Отрицателни хроно- и инотропни ефекти

В.) Артериоларна вазодилатация;

20. Основният елемент на антихипертензивното действие на бета-блокерите:

Изберете един отговор.

а.) Отрицателни хроно- и инотропни ефекти

B.) Артериоларна вазодилатация;

В.) Венозна вазодилатация;

D.) Блокада на рецепторите за ангиотензин II;

21. За лечение на артериална хипертония НЕ се използва:

Изберете един отговор.

а.) фуроземид

B.) спиронолактон;

C.) хидрохлоротиазид;

D.) манитол;

22. За системно лечение на артериална хипертония НЕ се прилагат:

Изберете един отговор.

а.) метопролол;

B.) лосартан;

В.) фентоламин

D.) нифедипин;

23. Бронходилататорите от групата на симпатикомиметиците включват:

Изберете един отговор.

а.) изадрин

Б.) ефедрин

В.) салбутамол

24. Негликозидните кардиотонични лекарства включват всички лекарства, С ИЗКЛЮЧЕНИЕ на:

Изберете един отговор.

а.) милринон

Б.) строфантин (уабаина);

В.) допамин;

D.) добутамин;

25. Антикоагуланти с непряко действие включват:

Изберете един отговор.

а.) хирудин

B.) Натриев хидроцитрат

В.) фраксипарин

D.) варфарин

26. Сърдечните гликозиди (CG) включват всички изброени по-долу лекарства, С ИЗКЛЮЧЕНИЕ на:

Изберете един отговор.

а.) дигоксин;

B.) добутамин;

В.) дигитоксин;

D.) строфантин

27. Каква комбинация от диуретици е рационална?

Изберете един отговор.

а.) Фуроземид + манитол

B.) Манитол + урея

В.) Дихлотиазид + триамтерен

D.) Фуроземид + етакринова киселина

28. Какви препарати от нитроглицерин се използват от пациенти с ангина пекторис за облекчаване на пристъпите?

Изберете един отговор.

а.) нитроглицерин в мехлем

Б.) нитроглицерин в сублингвални таблетки;

В.) нитроглицерин в микродраже (сустак);

D.) нитроглицерин в разтвор за венозно приложение;

29. Какви признаци на предозиране на SG са животозастрашаващи?

Изберете един отговор.

а.) умора, мускулна слабост

Б.) диспептични разстройства;

В.) зрителни нарушения;

Г.) камерни тахиаритмии;

30. Какви лекарства се използват за лечение на коронарна артериална болест?

Изберете един отговор.

а.) всичко по-горе

Б.) антиатеросклеротични средства;

В.) антитромботични средства;

D.) кардиопротективни средства;

31. Какво средство може да се използва за спиране на чревни спазми (колики)?

Изберете един отговор.

а.) метамизол (аналгин);

B.) метоклопрамид;

В.) дротаверин (no-shpa).

D.) морфин;

E.) магнезиев сулфат;

32. Кои от антиацидите могат да причинят алкалоза, ако се използват системно?

Изберете един отговор.

а.) магнезиев трисиликат;

Б.) алуминиев хидроксид;

В.) калциев глюконат;

D.) натриев бикарбонат

E.) магнезиев оксид;

33. Какъв антиеметик се използва при рефлукс, пареза на стомаха?

Изберете един отговор.

а.) ондансетрон (зофран);

Б.) хлорпромазин (хлорпромазин);

C.) метоклопрамид;

D.) дифенхидрамин (дифенхидрамин);

E.) перфеназин хидрохлорид (етаперазин)

34. Какъв диуретик може да причини загуба на слуха?

Изберете един отговор.

а.) спиронолактон

B.) Дихлотиазид

В.) манитол

D.) фуроземид

35. Какво лекарство се използва за подобряване на контрактилната активност на миометриума по време на раждане?

Изберете един отговор.

а.) ергометринов малеат

B.) Атропин сулфат

В.) окситоцин

D.) папаверин

36. Какво лекарство се използва за спиране на кървене от матката?:

Изберете един отговор.

а.) ергометринов малеат

B.) Атропин сулфат

В.) фенотерол

D.) простагландин F-2a

37. Кое лекарство принадлежи към антикоагуланти с директно действие?

Изберете един отговор.

а.) фибринолизин

Б.) фитоменадион

В.) Хепарин

D.) Варфарин

38. Какво лекарство се използва само за предотвратяване на повръщане, причинено от болест на пътуването (морска болест)?

Изберете един отговор.

а.) метоклопрамид (церукал);

B.) перфеназин хидрохлорид (етаперазин)

В.) дипразин (пиполфен);

D.) "Aeron";

E.) ондансетрон (зофран);

39. Какво лекарство намалява контрактилитета на миометриума?

Изберете един отговор.

а.) фенотерол

Б.) питуитрин

C.) простагландин F-2a

D.) Папаверин

40. Механизмът на бронходилататорното действие на атропина е свързан с:

Изберете един отговор.

а.) директно миотропно действие върху гладката мускулатура на бронхите

B.) блокада на m-холинергичните рецептори на бронхиалната гладка мускулатура

В.) възбуждане на В2-адренорецепторите

41. Механизъм на действие на бримковите диуретици (фуроземид и др.):

Изберете един отговор.

а.) Увеличете осмотичното налягане на течността в лумена на тубулите

Б.) Намаляване на реабсорбцията на натрий, хлорид и калий в дебелата част на възходящия крайник на бримката на Хенли

в.) Повишаване на гломерулната филтрация

D.) блокират карбанхидразата

42. Механизъм на действие на тиазидните диуретици?

Изберете един отговор.

а.) Увеличете осмотичното налягане на течността в тубулите на нефрона

B.) Повишаване на гломерулната филтрация

В.) блокират алдостероновите рецептори

г.) ​​Намаляване на реабсорбцията на натрий, хлорид в дисталните тубули

43. Механизмът на отхрачващо действие на препаратите от термопсис се осъществява поради:

Изберете един отговор.

а.) дразнене на рецепторите на стомаха и рефлекторно увеличаване на секрецията на бронхиалните жлези

б.) директно стимулиране на секрецията на бронхиалните жлези

в.) втечняване на храчките по време на деполимеризацията на протеина

44. Най-подходящото показание за предписване на СГ е:

Изберете един отговор.

а.) нестабилна стенокардия;

б.) ХСН с тежка брадикардия;

в.) ХСН с множество камерни екстрасистоли

г.) ​​ХСН с предсърдно мъждене;

45. Обща характеристика на всички AAS (с изключение на сърдечните гликозиди), използвани за лечение на тахиаритмии:

Изберете един отговор.

а.) Забавете бързата деполяризация

б.) Бавна реполяризация

в.) Ускоряване на реполяризацията

г.) ​​Намален автоматизъм

46. ​​​​Основното свойство на хепарина е:

Изберете един отговор.

а.) Кумулативен

б.) ефективен, когато се приема през устата

в.) Действието се развива след 18-24 часа

d.) Забавя съсирването на кръвта "in vivo" и "in vitro"

47. Характеристики на действието на лидокаин:

Изберете един отговор.

а.) Забавя бързата деполяризация

б.) Ускорява реполяризацията

в.) Забавя проводимостта

г.) ​​Повишава кръвното налягане

48. Маркирайте антитромбоцитния агент - COX инхибитор:

Изберете един отговор.

а.) варфарин

б.) фитоменадион

в.) ацетилсалицилова киселина

г.) ​​Натриев хидроцитрат

49. Маркирайте коагуланта с пряко действие:

Изберете един отговор.

а.) фитоменадион

б) тромбин

в.) апротинин

г.) ​​Хепарин

50. Обърнете внимание на страничния ефект на Eufilin:

Изберете един отговор.

а.) респираторна депресия

б.) повишена нужда от кислород на миокарда

в.) повишаване на кръвното налягане

51. Маркирайте лекарството, свързано с хидроуретиците:

Изберете един отговор.

а. 2) индапамид

b. 3) манитол

° С. 1) Дихлотиазид

д. 4) фуроземид

52. Маркирайте лекарството, свързано със салуретиците:

Изберете един отговор.

а.) Урея

б.) Манитол

в.) Демеклоциклин

г.) ​​Фуроземид

53. Маркирайте лекарството, което подобрява изтичането на жлъчка (холекинетик):

Изберете един отговор.

а.) "Холензим";

б.) дехидрохолева киселина;

в.) дротаверин (no-shpa);

г.) ​​магнезиев сулфат;

д.) атропин;

е.) аминофилин (еуфилин)

54. Маркирайте лекарството-холесекретик от растителен произход:

Изберете един отговор.

а.) Магнезиев сулфат;

б.) осалмид (оксафенамид);

в.) "Алохол";

г.) ​​Холензим

д.) дротаверин (no-shpa);

55. Маркирайте лаксатив за спешно прочистване на червата (подготовка за медицински или диагностични процедури):

Изберете един отговор.

а.) лактулоза;

б.) магнезиев сулфат;

в.) Изафенин;

г.) ​​глицеринови супозитории;

д.) фенолфталеин

56. Маркирайте средствата за заместителна терапия при хроничен панкреатит:

Изберете един отговор.

а.) контричен;

б.) пентагастрин

c.) мизопростол;

г.) ​​атропин;

д.) панкреатин;

57. Маркирайте агента, който намалява секрецията на солна киселина в стомаха:

Изберете един отговор.

а.) омепразол;

б.) натриев бикарбонат;

в.) алуминиев хидроксид

г.) ​​хистамин;

д.) пентагастрин

58. Защо коронарните литици (например дипиридамол) могат да причинят кражба на миокарда?

Изберете един отговор.

а.) тонизира коронарните съдове;

б.) повишават контрактилитета на миокарда

в.) преразпределят кръвния поток към здравите съдове в ущърб на исхемичната зона на миокарда;

г.) ​​разширяват съдовете на системното кръвообращение;

59. Използва се лекарство, което намалява бронхиалната реактивност от групата на НА при бронхиална астма:

Изберете един отговор.

а.) бекламетазон дипропионат

б.) кромолин натрий

в.) ипратропиев бромид

60. Лекарството по избор за спиране на бронхоспазъм е:

Изберете един отговор.

а.) изадрин

б) салбутамол

в.) атропин

61. Какви заболявания са диуретиците, използвани за планирана терапия:

Изберете един отговор.

а.) остро отравяне

б.) Мозъчен оток

в.) хипертония

г.) ​​Белодробен оток

62. При белодробен оток за намаляване на налягането в белодробното кръвообращение се прилагат:

Изберете един отговор.

а.) блокери на ганглии

б.) вдишване на кислород

в.) дихателни стимуланти

63. При белодробен оток вдишването на разтвор на етилов алкохол се използва за:

Изберете един отговор.

а.) противопенно действие

б.) наркотичен ефект

в.) дехидратиращ ефект

64. Антиаритмичното действие на сърдечните гликозиди се дължи на:

Изберете един отговор.

а.) Намалена сила на сърдечните контракции

б.) Забавяне на проводимостта

в.) Намален автоматизъм

г.) ​​Намалена възбудимост

65. Антитусивно средство, което потиска кашличния рефлекс и блокира възбудимостта на чувствителните окончания в дихателните пътища е:

Изберете един отговор.

а.) тусупрекс

б) кодеин

в.) либексин

66. Смесен стимулиращ ефект върху дихателния център има:

Изберете един отговор.

а.) кофеин

б.) никетамид (кордиамин)

в.) цититон

67. Специфичен нежелан страничен ефект на инхибитори на ангиотензин-конвертиращия ензим:

Изберете един отговор.

а.) суха кашлица

б.) агранулоцитоза;

в.) ринорея;

г.) ​​анорексия;

68. Средства за лечение на брадиаритмии

Изберете един отговор.

а.) Верапамил

в.) Лидокаин

г.) ​​Атропин

69. Средства за лечение само на камерни тахиаритмии

Изберете един отговор.

а.) Пропафенон

б.) Прокаинамид (новокаинамид)

в.) Лидокаин

г.) ​​Верапамил

70. Средства за лечение само на суправентрикуларни тахиаритмии

Изберете един отговор.

а.) Лидокаин

б.) Прокаинамид (новокаинамид)

в.) Верапамил

г.) ​​Пропафенон

71. Посочете най-честата нежелана реакция на нитратите:

Изберете един отговор.

а.) образуване на метхемоглобин;

б.) главоболие;

в.) намаляване на тонуса на жлъчния канал

d.) инхибиране на тромбоцитната агрегация;

72. Какви са показанията за назначаване на метоклопрамид?

Изберете един отговор.

а.) диария;

б.) ниска киселинност;

в.) свръхкиселинност;

г.) ​​кинетози (морска, въздушна болест);

д.) гадене, повръщане.

Преглед:

Предмет:„Лекарства, засягащи

на централната нервна система"

Тестови задачи

1. Какво е вярно за ACK?

Изберете един отговор.

а.) не се използва при артрит;

б.) най-малко улцерогенен;

в.) да не се използва при деца под 14 години с температура;

d.) използван като антиагрегант в дози, по-високи от аналгетичните

2. Какво НЕ е противопоказание за употребата на опиоидни аналгетици?

Изберете един отговор.

а.) респираторна депресия;

б.) миокарден инфаркт;

в.) черепно-мозъчна травма

г.) ​​остра коремна болка с неясен произход;

3. Какво се използва за възстановяване на дишането в случай на предозиране на хероин (морфин)?

Изберете един отговор.

а.) налоксон;

б.) кислород;

c.) трамадол;

г.) ​​налтрексон

4. Какво е характерно за антипиретичния ефект на ненаркотичните аналгетици?

Изберете един отговор.

а.) НА причиняват хипотермия чрез потискане на производството на топлина;

б.) назначаването е задължително при субфебрилна температура;

c.) НА намалява треската чрез увеличаване на разсейването на топлината

г.) ​​присъщ е на всички НА в дози значително по-високи от аналгетика;

5. Какво е характерно за пиразолоновите производни (метамизол (аналгин), фенилбутазон (бутадион))?

Изберете един отговор.

а.) използвани като антиагреганти при коронарна артериална болест

б.) се използват за продължителна терапия на артрит;

в.) са хематотоксични;

г.) ​​няма противовъзпалителен ефект;

6. Какво е характерно за противовъзпалителното действие на НСПВС?

Изберете един отговор.

а.) подобряване на качеството на живот на пациентите чрез временно намаляване на симптомите на артрит;

б.) излекуване на артрит с пълен курс на лечение;

в.) инхибират всички фази на възпалението;

г.) ​​противовъзпалителен ефект поради инхибиране на синтеза на левкотриени7. Каква е причината за смъртта при предозиране на опиоидни аналгетици?

Изберете един отговор.

а.) бронхоспазъм;

б.) белодробен оток;

в.) спиране на дишането;

г.) ​​сърдечен арест

8. Екстрапирамидни двигателни нарушения - типичен нежелан страничен ефект:

Изберете един отговор.

а.) Клозапин

б.) Халоперидол

в.) Оланзапин

г.) ​​Рисперидон

9. Какво лекарство се използва при епилептичен статус?

Изберете един отговор.

а.) дифенхидрамин (дифенхидрамин);

б.) диазепам;

в.) етосуксимид

10. Какъв ефект на опиоидните аналгетици може да бъде опасен дори при еднократна употреба на фона на силна болка

Изберете един отговор.

а.) спазми в стомашно-чревния тракт;

б.) еуфория;

в.) запек

г.) ​​респираторна депресия;

11. Какъв ефект на опиоидните аналгетици ограничава широкото им използване?

Изберете един отговор.

а.) аналгетик;

б.) успокоително;

° С. 3) еуфоричен;

д. 4) спазматичен

12. Кофеин:

Изберете един отговор.

а.) Тонизира дихателния и вазомоторния център

б.) Свива коронарните съдове

в.) Разширява мозъчните съдове

13. Моклобемид, в сравнение с имипрамин, има по-силен:

Изберете един отговор.

а.) Психостимулиращо действие

б.) Психоседативно действие

в.) Алфа-адренергично блокиращо действие

г.) ​​М-антихолинергично действие

14. Нимезулид и целекоксиб - селективни COX-2 инхибитори - се различават от неселективните (ASA, диклофенак и др.):

Изберете един отговор.

а.) по-ефективен;

б.) по-малка честота на гастропатия;

в.) по-малко алергенни;

d.) значително по-малка тежест на всички "PG-зависими" странични ефекти

15. Основното показание за употребата на опиоидни (наркотични) аналгетици

Изберете един отговор.

а.) Травматична и висцерална болка с висок интензитет

б.) травматична и висцерална болка с умерен интензитет;

в.) невралгия;

г.) ​​остеоалгия;

16. Обърнете внимание на нежеланите реакции, общи за NA / НСПВС („COX- и PG-зависими“):

Изберете един отговор.

а.) пристрастяване, наркотична зависимост;

б.) тежест, потискане на дишането;

в.) алергични реакции, левкопения

г.) ​​гастропатия, кървене;

17. Отбележете правилното твърдение за кеторолак:

Изберете един отговор.

а.) е ефективен само при умерена болка;

б.) използва се за продължителна терапия на артрит;

в.) не се използва повече от 5-7 дни поради нефротоксичност

г.) ​​хепатотоксичен;

18. При паркинсонизъм се прилагат:

Изберете един отговор.

а.) фенитоин (дифенин);

б.) карбамазепин;

в.) леводопа

19. Антиеметичен ефект има:

Изберете един отговор.

а.) Хлорпромазин

б.) Буспирон

в.) Зопиклон

г.) ​​Диазепам

20. Антиконвулсивен ефект има:

Изберете един отговор.

а.) Халоперидол

б.) Диазепам

в.) Буспирон

г.) ​​Хлорпромазин

21. Психостимулант от групата на метилксантините:

Изберете един отговор.

а.) Амфетамин

б.) Кофеин

в.) Моклобемид

г.) ​​Пирацетам

д.) Имипрамин

22. Специфично средство за лечение на остро отравяне с бензодиазепинови производни на транквиланти:

Изберете един отговор.

а.) Зопиклон

б.) Флумазенил

в.) Кофеин

г.) ​​Пирацетам

д.) Феназепам

23. Транквилизатор с най-дълъг полуживот (T1 / 2> 48 часа):

Изберете един отговор.

а.) Диазепам

б.) Оксазепам

в.) Лоразепам

г.) ​​Медазепам

д.) Мидазолам

24. Трицикличен антидепресант:

Изберете един отговор.

а.) Кофеин

б.) Амитриптилин

в.) Флуоксетин

г.) ​​Пирацетам

25. Посочете характеристиките на аналгетичния ефект на NA:

Изберете един отговор.

а.) са ефективни при умерена артралгия, миалгия, цефалгия;

б.) премахване на болка от всякакъв интензитет;

в.) по-ефективен от наркотичните аналгетици при силна травматична и висцерална болка;

г.) ​​продължителното използване развива толерантност

26. Флуоксетин срещу амитриптилин ::

Изберете един отговор.

а.) По-малко токсичен

б.) Има по-силен М-антихолинергичен ефект

c.) По-клинично ефективен

г.) ​​Има по-силен седативен ефект

27. Каква е разликата между хипнотичните производни на барбитуровата киселина и производните на бензодиазепина?

Изберете един отговор.

а.) Изразено централно мускулно релаксиращо действие

б.) повече нарушаване на структурата на съня;

в.) слаба индукция на микрозомални чернодробни ензими;

28. Как се различават частичните агонисти и агонистите-антагонисти на опиоидните рецептори (пентазоцин, бупренорфин) от пълните агонисти (морфин)?

Изберете един отговор.

а.) по-силно спазматично действие;

б.) по-малко пристрастяване;

в.) възможно е ректално приложение

г.) ​​по-силно потискане на дишането;

29. Алостеричен активатор на GABA-A рецептори:

Изберете един отговор.

а.) Баклофен

б.) Диазепам

в.) Буспирон

г.) ​​Амизил

30. Антидепресант, селективен MAO-A инхибитор:

Изберете един отговор.

а.) Моклобемид

б.) Пирацетам

в.) Флуоксетин

г.) ​​Имипрамин

д.) Амитриптилин

е.) Кофеин

31. Антидепресант, селективен инхибитор на обратното захващане на серотонина:

Изберете един отговор.

а.) Пирацетам

б.) Флуоксетин

в.) Кофеин

г.) ​​Имипрамин

д.) Амитриптилин

32. Антиманиакален ефект няма:

Изберете един отговор.

а.) Халоперидол

б.) Литиев карбонат

в.) Диазепам

г.) ​​Трифтазин

33. Антипсихотично средство от групата на фенотиазиновите производни:

Изберете един отговор.

а.) Рисперидон

б.) Оланзапин

в.) Хлорпромазин

г.) ​​Клозапин

д.) Халоперидол

34. Атипичен антипсихотик:

Изберете един отговор.

а.) Флуорфеназин

б.) Халоперидол

в.) Клозапин

г.) ​​Хлорпромазин

д.) Трифтазин

35. Основният психотропен ефект на пирацетам:

Изберете един отговор.

а.) Анксиолитик

б.) Седация

в.) Мнемотропен

г.) ​​Психостимулант

36. За НСПВС всички изброени по-долу лекарствени взаимодействия са характерни, ОСВЕН:

Изберете един отговор.

а.) кодеинът намалява аналгетичния ефект на NA или НСПВС;

б.) НСПВС отслабват ефекта на диуретиците и редица антихипертензивни средства;

в.) алуминий-съдържащите антиациди намаляват бионаличността на НСПВС

г.) ​​седативите усилват аналгетичния ефект на НСПВС;

37. Дневен успокоител:

Изберете един отговор.

а.) Феназепам

б.) Зопиклон

в.) Медазепам

г.) ​​Диазепам

д.) Аминазин

38. Вентрикуларни тахиаритмии потенциален нежелан страничен ефект:

Изберете един отговор.

а.) Типични антипсихотици

б.) Трициклични антидепресанти

c.) Селективни антидепресанти инхибитори на обратното захващане на серотонина

г.) ​​Бензодиазепинови транквиланти

д.) Атипични антипсихотици

39. Симптомите на остро отравяне със сънотворни включват:

Изберете един отговор.

а.) възбуда, повишаване на кръвното налягане;

б.) кома, респираторна депресия, хипоксия;

в.) повишаване на температурата, повишена рефлекторна възбудимост

40. Какви опиоидни аналгетици са противопоказани при инфаркт на миокарда?

Изберете един отговор.

а.) пентазоцин, буторфанол;

б.) морфин, промедол;

в.) фентанил, налбуфин

41. Какви лекарства могат да се използват при спастичност на скелетните мускули?

Изберете един отговор.

а.) стрихнин, никетамид (кордиамин), бемегрид

б.) баклофен, диазепам, мидокалм;

в.) прозерин, галантамин, физостигмин;

42. Кое твърдение относно ацетаминофен (парацетамол) НЕ е вярно?

Изберете един отговор.

а.) НСПВС по избор за артрит

б.) гастротоксични;

в.) няма антитромбоцитен ефект;

г.) ​​антипиретик на избор при вирусни инфекции при деца;

43. Кой високоактивен опиоиден аналгетик е предпочитан за обезболяване при краткотрайни болезнени манипулации/операции?

Изберете един отговор.

а.) морфин;

б) пентазоцин

c.) фентанил;

г.) ​​промедол;

44. Кое лекарство принадлежи към антиепилептичните лекарства?

Изберете един отговор.

а.) натриев валпроат;

б) леводопа;

в.) циклодол

45. Кое лекарство принадлежи към сънотворните?

Изберете един отговор.

а.) циклодол;

б.) зопиклон;

в.) фенитоин (дифенин);

г.) ​​леводопа

46. ​​​​Какво лекарство е за предпочитане за анестезия на първия етап на раждането?

Изберете един отговор.

а.) кодеин

б.) метамизол (аналгин);

в.) морфин;

г.) ​​тримеперидин (промедол);

а.) медни препарати

б.) Фосфор

в.) живачни съединения

г.) ​​железни съединения

2. За кое лекарствено вещество първичната фармакологична реакция се дължи на влиянието върху процесите на ДНК транскрипция:

Изберете един отговор.

а.) инсулин;

б.) бензилпеницилин

в.) хепарин;

г.) ​​преднизолон;

3. За кое лекарство първичната фармакологична реакция се дължи на намаляване на пропускливостта на волтаж-зависимите йонни канали:

Изберете един отговор.

а.) дигитоксин;

б.) лидокаин;

в.) ропина;

г.) ​​фуроземид

4. За кое лекарство първичната фармакологична реакция се дължи на намаляване на пропускливостта на медиатор-зависимите (хемочувствителни) йонни канали:

Изберете един отговор.

а.) лидокаин;

б) пипекуроний

в.) парацетамол;

г.) ​​верапамил;

5. За кое лекарство първичната фармакологична реакция се дължи на инхибирането на ензимната активност:

Изберете един отговор.

а.) лидокаин;

б.) адреналин;

в.) прозерин

г.) ​​атропин;

6. За кое лекарствено вещество първичната фармакологична реакция се дължи на инхибиране на процеса на улеснена дифузия:

Изберете един отговор.

а.) адреналин;

б.) дихлотиазид.

c.) дигоксин;

г.) ​​диазепам;

7. За да намалите концентрацията на отрова в кръвта и тъканите, използвайте:

Изберете един отговор.

а.) лаксативи

б.) химически противоотрови

в.) адсорбенти

г.) ​​функционални антидоти

8. За да се отстрани неабсорбираната отрова от стомаха, последният се измива с вода с добавяне на:

Изберете един отговор.

а.) разтвор на атропин

б.) натриев сулфат

в.) метилтионинов хлорид (метиленово синьо)

г.) ​​активен въглен

9. Комплексоните включват:

Изберете един отговор.

а.) Пентацин

б.) налоксон

в.) натриев тиосулфат

г.) ​​Пентамин

10. Какви средства се използват за стимулиране на дихателния център:

Изберете един отговор.

а.) никетамид (кордиамин); бемегрид; сулфокамфокаин;

б.) морфин; фентанил; тримеперидин (промедол)

в.) епинефрин (адреналин); фенилефрин (мезатон); норепинефрин (норепинефрин)

г.) ​​дротоверин (но-шпа); метацин; папаверин;

11. Натриевият тиосулфат образува нискотоксични роданидни съединения в случай на отравяне:

Изберете един отговор.

а.) хероин

б) цианиди

в.) атропин

г.) ​​сърдечни гликозиди

12. Основните цели на лечението на остро отравяне са всички, с изключение на:

Изберете един отговор.

а.) намаляване на концентрацията на отрова в кръвта и тъканите

б.) намаляване на по-нататъшното усвояване на отровата

в.) нормализиране на функциите на жизненоважни органи и системи

г.) ​​забавяне на метаболизма на отровата

13. Функционалният антидот на морфина е:

Изберете един отговор.

а.) Дифенхидрамин (дифенхидрамин)

б) атропин

в.) налоксон

г.) ​​бемережа

14. Химически антидот при предозиране на хепарин е:

Изберете един отговор.

а.) фитоменадион

б.) калциев хлорид

в.) протамин сулфат

г.) ​​димеркапрол (унитиол)

15. Етиловият алкохол променя метаболизма на отровата в случай на отравяне:

Изберете един отговор.

а.) метилов алкохол

б) атропин

в.) морфин

г.) ​​арсенови препарати


1. Как се нарича разделът на фармакологията, който изучава абсорбцията, разпределението, биотрансформацията и екскрецията на лекарства?

Фармакокинетика.

Фармакодинамика.

2. Как се нарича разделът на фармакологията, който изучава видовете действие на лекарствата, фармакологичните ефекти, механизма на действие?

Фармакодинамика.

Фармакокинетика.

3. Основният механизъм на абсорбция на лекарството в стомашно-чревния тракт:

активен транспорт.

Улеснена дифузия.

Пасивна дифузия през клетъчните мембрани.

Пиноцитоза.

4. Основното място на абсорбция на лекарството е слабите основи:

Тънко черво.

5. Основното място на абсорбция на лекарството е слабите киселини:

Тънко черво.

6. Какъв начин на приложение на лекарството осигурява 100% бионаличност?

Интрамускулно.

Ректално.

Интравенозно.

През устата.

7. Как ще се промени абсорбцията на лекарства - слаби киселини с намаляване на киселинността на стомашния сок?

ще нарастне.

ще намалее.

8. Как ще се промени абсорбцията на лекарства - слаби основи с намаляване на киселинността на стомашния сок?

ще нарастне.

ще намалее.

9. Чрез пасивна дифузия веществата лесно се транспортират през биологични мембрани:

Липофилни.

Полярен.

Хидрофилен.

10. Ентерален начин на приложение на лекарството:

Интрамускулно.

Вдишване.

Под езика.

Интравенозно.

11. Парентерален начин на приложение на лекарството:

През устата.

В ректума.

Подкожно.

Под езика.

12. Къде се извършва абсорбцията на повечето лекарства?

В месеца.

В стомаха

В тънките черва.

В дебелото черво.

13. Можете да въведете интравенозно:

Маслени разтвори.

неразтворими съединения.

Осмотично активни съединения.

Микрокристални суспензии.

неразтворими съединения.

14. Какви функционални промени в тялото причиняват сърдечни гликозиди при сърдечна недостатъчност?

Възбуда.

Потисничество.

Тонизиране.

Спокоен.

15. Каква функционална промяна в тялото причинява лекарство, което понижава кръвното налягане при артериална хипертония?

Възбуда.

Потисничество.

Тонизиране.

Спокоен.

16. Как се нарича натрупването на лекарство в тялото при многократното му инжектиране?

функционална кумулация.

Сенсибилизация.

материално натрупване.

Тахифилаксия.

17. Толерантността е:

Алергична реакция на тялото към многократно приложение на лекарството.

Намаляване на фармакологичния ефект при многократно приложение на лекарството.

Непреодолимо желание да вземете лекарството отново.

18. Намаляването на ефекта от прилагането на лекарства на кратки интервали е:

Тахифилаксия.

Идиосинкразия.

Сенсибилизация.

Пристрастяване.

19. Страничен ефект, който може да възникне самопри многократно приложение на лекарства:

Идиосинкразия.

Тератогенно действие.

мутагенно действие.

Пристрастяване.

20. Страничен ефект, който може да възникне самопри употреба на психотропни лекарства:

Идиосинкразия.

Пристрастяване.

Пристрастяване.

Сенсибилизация.

21. Определете вида на лекарственото взаимодействие: пациент с отравяне с мускарин е подложен на стомашна промивка със суспензия от активен въглен:

Комбинирана синергия.

химически антагонизъм.

конкурентен антагонизъм.

физически антагонизъм.

22. Мутагенното действие е:

23. Тератогенният ефект е:

Увреждане на генетичния апарат на зародишната клетка.

Нарушаване на диференциацията на феталните тъкани, причинявайки различни аномалии.

Страничен ефект, който се проявява през първите 12 седмици след оплождането и причинява смъртта на ембриона.

24. Ембриотоксичното действие е:

Увреждане на генетичния апарат на зародишната клетка.

Нарушаване на диференциацията на феталните тъкани, причинявайки различни аномалии.

Страничен ефект, който се проявява през първите 12 седмици след оплождането и причинява смъртта на ембриона.

КЛИНИЧНА ФАРМАКОЛОГИЯ

001. Как се променя обемът на разпределение на мастноразтворимите лекарства

при пациенти със затлъстяване?

а) намалява

в) не се променя

г) не се променя или се увеличава

г) се увеличава

002. Как се променя биотрансформацията на лекарствата на фона на тютюнопушене и пиене на алкохол?

а) намалява

б) намалява или не се променя

в) не се променя

г) не се променя или се увеличава

г) засилване

003. От какъв основен фактор зависи биоеквивалентността на лекарството?

а) фармакодинамични характеристики на лекарствата

б) физични и химични характеристики

в) лекарствена форма

г) технология на производство

д) състоянието на тялото на пациента

004. Какви странични ефекти на лекарствата не зависят от дозата?

а) свързани с фармакологичните свойства на лекарствата

б) токсични усложнения,

поради абсолютно или относително предозиране

в) вторични ефекти, дължащи се на нарушението

имунобиологични свойства на организма

г) имунологични реакции от незабавен и забавен тип

д) синдром на отнемане

005. Какво е усложнението при новородените

може да предизвика въвеждането на магнезиев сулфат на бременни жени преди раждането?

а) развитие на невромускулни блокади и летаргия

б) респираторна депресия

в) тромбоцитопения

г) хепатотоксичен ефект

д) недохранване

006. Какви усложнения при новородени

може да предизвика назначаването на бременни жени -адренергични блокери?

а) загуба на слуха

б) увреждане на кожата

в) недохранване на плацентата и плода

г) преждевременно затваряне на ductus arteriosus

д) хеморагичен синдром

007. Приложение на кои антимикробни лекарства

най-безопасен по време на бременност?

а) аминогликозиди

б) котримоксазол

в) нитрофурани

г) пеницилини

д) флуорохинолони

008. Посочете антибиотик, който няма антипсевдомонална активност:

а) карбеницилин

б) ампиокс

в) тикарцилин

д) азлоцилин

г) цефтазидим

009. Пациент с пневмония на антибиотично лечение,

започна да се оплаква от световъртеж,

нестабилност и залитане при ходене.

Какъв антибиотик може да причини тези симптоми?

а) ампицилин

б) цефоперазон

в) гентамицин

г) еритромицин

д) линкомицин

010. Посочете антибиотика, който е избраното лекарство

при лечение на инфекции, причинени от стафилокок. ауреус:

а) пеницилин

б) гентамицин

в) азитромицин

г) хлорамфеникол

д) амоксицилин-клавуланат

011. Посочете антибиотика, който е избраното лекарство

при лечение на инфекции,

причинени от метицилин-резистентен стафилокок:

а) линкомицин

б) еритромицин

в) ванкомицин

г) пеницилин

д) оксацилин

012. Посочете антибиотика,

а) ампицилин

б) гентамицин

в) цефоперазон

г) метранидазол

д) тетрациклин

013. Активен срещу атипични патогени

(микоплазма, хламидия, легионела):

а) гентамицин

б) еритромицин

в) ампиокс

г) хлорамфеникол

д) клиндамицин

014. Котримоксазол е лекарство на избор при лечение на:

а) пневмоцистна пневмония при имунокомпрометирани пациенти

б) дифтерия

в) холангит

г) пневмококова пневмония

д) амебна дизентерия

015. Какъв антибиотик е противопоказан за пациенти,

прием на мускулни релаксанти или миастения гравис?

а) ампицилин

б) гентамицин

в) еритромицин

г) линкомицин

д) ципрофлоксацин

016. Болен от бронхиална астма,

постоянно приемане на перорални инхалации на преднизон, теопек, беротек,

поради свързана бронхопулмонална инфекция

Предписани са еритромицин и бромхексин.

На третия ден от лечението пациентът разви главоболие, тревожност,

раздразнителност, сърцебиене, усещане за прекъсване на сърцето,

понижаване на кръвното налягане, треска, гадене, повръщане.

С токсичния ефект на кое лекарство са свързани тези симптоми?

а) преднизон

б) теопек

в) еритромицин

г) беротек

д) бромхексин

017. Концентрацията на теофилин в кръвта се намалява от всички тези лекарства,

а) фенобарбитал

б) рифампицин

в) карбамазипин

г) нифедипин

д) фенитоин

018. Средната терапевтична концентрация на теофилин в плазмата е:

а) 10-20 mcg/ml

б) 25-30 mcg/ml

в) 30-35 mcg/ml

г) 35-40 mcg/ml

д) 5-10 mcg/ml

019. С комбинираното действие на теофилин и циметидин ефектът на еуфилин:

а) засилва се

б) нараства или не се променя

в) намалява

г) намалява или не се променя

г) не се променя

020. Продължителност на действие на ксантините:

а) 1-2 часа

б) 2-3 часа

в) 3-4 часа

г) 6-8 часа

д) 10-12 часа

021. Какъв е механизмът на действие на 2-адренергичните агонисти?

а) инхибиране на фосфодиестераза

б) инхибиране на дегранулацията на мастоцитите

в) блокиране на хистаминовите рецептори

г) инхибиране на действието на левкотриени върху дихателните пътища

д) активиране на аденилатциклазата, увеличаване на образуването на сАМР

022. Посочете началото, максималното действие и продължителността на действие

фенотерол (беротек):

а) незабавно, 10 минути, 6 часа

б) 15 минути, 30 минути, 6 часа

в) 2-3 минути, 20 минути, 2,5 часа

г) 5-10 минути, 30 минути, 6 часа

д) 30-40 секунди, 20 минути, 3-5 часа

023. Какви са най-честите нежелани реакции?

инхалаторни форми на глюкокортикоиди:

а) развитие на остеопороза

б) хиперкортицизъм

в) кандидоза на устната кухина и фаринкса

г) артериална хипертония

024. За разлика от беклометазон пропионат, будезонидът има:

а) по-голям афинитет към рецепторите в белите дробове,

претърпява активна биотрансформация в черния дроб

при първо преминаване

б) инхибира в по-голяма степен образуването на хидрокортизон

в) по-често води до развитие на хипергликемия

г) по-често причинява обостряне на бронхо-белодробна инфекция

д) няма разлика между лекарствата

025. Посочете дългодействащ 2-агонист:

а) салбутамол

б) тербуталин

в) фенотерол

г) орципреналин сулфат

д) формотерол

026. Посочете лекарството, което има най-силен инхибиращ ефект

за стомашна секреция:

а) омепразол

б) циметидин

в) фамотидин

г) сукралфат

за лечение на рецидив на язва на дванадесетопръстника:

028. Пациент, страдащ от ревматоиден артрит,

продължително приемане на НСПВС.

Какво лекарство е показано за този пациент

за предотвратяване на язва?

а) сукралфат

б) гастроцепин

в) ранитидин

г) маалокс

д) мизопростол

029. При лечението на хеликобактериоза най-ефективното приложение е:

а) ранитидин

б) оксацилин

в) де-нола

г) де-нола + ампицилин (амоксицилин)

д) маалокс

030. Най-рационалният режим на предписване на антиациди

при пациенти с пептична язва:

а) 20 минути преди хранене

б) веднага след хранене

в) 20 минути след хранене и през нощта

г) един час след хранене и през нощта

д) независимо от приема на храна 4-5 пъти на ден

031. Какви антихипертензивни лекарства

действат предимно като постганглионарни адренергични блокери?

а) пентамин

б) клонидин

в) гуанетидин сулфат

г) анаприлин

д) хлорталидон

032. Какви антихипертензивни лекарства

повлияват неврохуморалните механизми на регулиране на кръвното налягане?

а) клонидин

б) каптоприл

в) миноксидил

г) гуанетидин

д) хидрохлоротиазид

033. Посочете механизмите на хипотензивното действие на veroshpiron:

а) намалена активност на плазмения ренин

б) блокада на адренергичните рецептори

в) намаляване на обема на циркулиращата течност

г) намаляване на общото периферно съпротивление

д) конкурентен алдостеронов антагонист

034. Бионаличността на нифедипин е ниска поради:

а) пресистемно елиминиране в черния дроб

б) ниска абсорбция

в) свързване с плазмените протеини

г) инактивиране в стомашно-чревния тракт

035. Посочете продължителността на хипотензивното действие на еднократна доза клонидин

при перорален прием:

а) 1-2 часа

б) 6-8 часа

в) 10-12 часа

г) 2-24 часа

д) до 3 дни

036. Облекчаването на неусложнена хипертонична криза трябва да започне:

а) с интрамускулно инжектиране на 1,0 ml 0,01% разтвор на клонидин

б) с 40 mg фуроземид перорално

в) с 10-20 mg нифедипин сублингвално

г) с 40 mg анаприлин вътре

д) с интравенозно 1,0 ml 0,5% разтвор на фентоламин

037. Каптоприл е противопоказан при пациенти:

а) цироза на черния дроб

б) с хронична бъбречна недостатъчност

в) с cor pulmonale

г) с диабет

д) с пептична язва

038. Посочете механизма на действие на нитроглицерина:

а) блокада на -адренергичните рецептори

б) спазмолитично, миотропно действие

върху гладката мускулатура на съдовата стена

в) блокиране на малки калциеви канали на клетъчната мембрана

г) повишена активност на α-рецепторите

съдовата стена на коронарните артерии

д) увеличава бавния поток на калций в клетката

039. Какви са противопоказанията за предписване на нитрати?

а) остър миокарден инфаркт

б) артериална хипертония

в) хипотония

г) брадикардия

д) атриовентрикуларна блокада

040. На какво се дължи антиангинозният ефект на -адренергичните блокери?

а) разширяване на коронарните съдове

б) намаляване на пост- и преднатоварването на сърцето

в) намалена работа на сърцето

г) централен механизъм на действие

д) повишена нужда от кислород на миокарда

041. Посочете -блокер със селективно действие:

а) скара се

б) тразикор

в) уиски

г) сектор

042. Каква е бионаличността на пропранолол, когато се приема перорално?

043. Каква е бионаличността на верапамил при перорален прием?

044. Посочете противопоказания за назначаването на нифедипин:

а) артериална хипертония

б) сърдечна недостатъчност

в) бронхиална астма

г) артериална хипотония

д) атриовентрикуларна блокада 2-ра степен

045. Посочете лекарството, принадлежащо към 1-ва група антиаритмични лекарства

(мембрано стабилизиращо действие):

а) лидокаин

б) изоптин

в) кордарон

г) хинидин

д) дифинин

046. Посочете лекарството, принадлежащо към 2-ра група антиаритмични лекарства

(местни анестетици):

а) мекситил

б) обзидан

в) уиски

г) кордарон

д) новокаинамид

047. Посочете лекарството, принадлежащо към 3-та група антиаритмични лекарства

(-блокери):

а) лидокаин

б) тразикор

в) кордарон

г) хинидин

д) изоптин

048. Посочете продължителността на действие на лидокаина:

а) 20 минути

б) 60 минути

в) 1,5-2 часа

д) 12 часа

049. Посочете полуживота на кордарон:

а) 4-6 часа

б) 1-2 часа

в) 20-24 часа

050. Посочете времето за достигане на максималната концентрация на изоптин

в плазмата при перорален прием:

а) 10 минути

б) 50 минути

в) 1,5-2 часа

г) 8 часа

д) 10 часа

051. Посочете местната упойка,

с най-изразен антиаритмичен ефект:

а) лидокаин

б) тримекаин

в) ксикаин

г) новокаин

д) мекситил

052. Развитието на толерантност към нитрати зависи главно от:

а) от пътя на приложение на нитрата

б) от момента на достигане на максимална концентрация в кръвта

в) продължителност на действие

г) комбинация с други лекарства

г) нито един от горните фактори

не влияе върху развитието на толерантност

053. За разлика от изосорбид динитрата, 5-изосорбид мононитрат:

а) не претърпява първично пресистемно елиминиране

при преминаване през черния дроб

б) не предизвиква развитие на толерантност

в) не предизвиква метхемоглобиния

г) не предизвиква главоболие

д) няма разлики между лекарствата

054. Назовете противовъзпалително средство с удължено действие:

а) аспирин

б) аналгин

в) пироксикам

г) индометацин

д) ортофен

055. При приемане на салицилати се наблюдава най-ниската им концентрация:

а) в бъбреците

б) в черния дроб

в) в миокарда

г) в белите дробове

г) в мозъка

056. Наименувайте глюкокортикоидни препарати с удължено действие:

а) преднизон

б) полкортолон

в) дексаметазон

г) кеналог

д) метилпреднизолон

057. Посочете страничния ефект на никотиновата киселина:

а) липодистрофия

б) хиперурикемия

в) рабдомиоза

г) зрително увреждане

д) бронхоспазъм

058. Посочете страничния ефект на секвестрантите на жлъчните киселини:

а) сърбеж по кожата

б) диария

в) запек

г) зрително увреждане

д) депресия

059. Какво антихистаминово лекарство е противопоказано

а) супрастин

б) пиполфен

в) дифенхидрамин

г) тавегил

д) фенкарол

060. Какво антихистаминово лекарство е противопоказано

при лечение на алергични реакции към прилагане на лекарства,

а) пиполфен

б) супрастин

в) дифенхидрамин

г) тавегил

д) фенкарол

061. Полуживотът на лекарствата е:

а) времето за достигане на максималната концентрация на лекарството в плазмата

б) времето, през което лекарството достига системното кръвообращение

в) времето, през което лекарството се разпределя в организма

г) времето, за което концентрацията на лекарството в плазмата намалява с 50%

д) времето, необходимо на половината от приложената доза да достигне до целевия орган

062. Терапевтичен индекс е:

а) терапевтичната доза на лекарството

б) съотношението на концентрацията на лекарството в орган или тъкан

към плазмената му концентрация

в) диапазонът между минимума и максимума

г) процент на несвързано с протеин лекарство

д) диапазон между минимум и максимум

терапевтични концентрации на лекарства

063. Конкурентните рецепторни лекарства включват:

а) нестероидни противовъзпалителни средства

б) -блокери

в) бримкови диуретици

г) нитрати

д) флуорохинолони

064. При предписване на следните лекарства

трябва да се вземе предвид функцията както на черния дроб, така и на бъбреците:

а) липофилни, образуващи неактивни метаболити

б) липофилни, образуващи активни метаболити

в) хидрофилен

г) хепатотоксичен

д) нефротоксичен

065. Избирателността на действието на лекарственото вещество зависи от:

а) от полуживота

б) по начина на приемане

в) от връзката с белтъка

г) върху обема на разпространение

д) върху дозата

066. Следните групи нежелани реакции са строго дозозависими:

а) фармацевтични

б) токсичен

в) алергични

г) мутагенен

д) синдром на отнемане

067. Избройте групите лекарства с тесен терапевтичен индекс:

а) блокери

б) пеницилини

в) сърдечни гликозиди

г) метилксантини

д) мощни диуретици

068. Лекарство на избор при наличие на атипични патогени

(микоплазма, хламидия) е:

а) еритромицин

б) метронидазол

в) гентамицин

г) карбеницилин

д) цефуроксим

069. Лекарства на избор при наличие на атипични патогени

(микоплазма, хламидия) са:

а) макролиди

б) пеницилини

в) аминогликозиди

г) цефалоспорини

д) сулфонамиди

070. Посочете антибактериално лекарство,

с най-висока антианаеробна активност:

а) еритромицин

б) ампицилин

в) тетрациклин

г) гентамицин

д) цефотетан

071. Чревна дисбактериоза

Причиняват всички от следните антибиотици с изключение на:

а) полусинтетични лекарства

б) тетрациклини

в) флуорохинолони

г) перорални цефалоспорини

д) макролиди

072. Следните антибактериални лекарства са нефротоксични,

а) гентамицин

б) карбеницилин

в) еритромицин

г) цефазолин

д) ванкомицин

073. Посочете антибактериалното лекарство,

не е активен срещу пневмококи:

а) азитромицин

б) пеницилин

в) цефтриаксон

г) ципрофлоксацин

д) хлорамфеникол

074. Изберете комбинация от антибактериални лекарства,

със синергия от действие и безопасност:

а) пеницилини + тетрациклини

б) пеницилини + цефалоспорини

в) пеницилини + макролиди

г) пеницилини + аминогликозиди

д) пеницилини + сулфонамиди

075. Добро проникване през кръвно-мозъчната бариера

следните антибактериални лекарства:

а) пеницилини

б) макролиди

в) тетрациклини

г) аминогликозиди

д) цефалоспорини

076. Лекарството по избор при лобарна пневмония е:

а) цефаклор

б) доксициклин

в) метицилин

г) цефотаксим

д) пеницилин

077. Лекарството на избор при фарингит е:

а) цефаклор

б) тетрациклин

в) цефтазидим

г) офлоксацин

д) пеницилин

078. Ново поколение макролидни антибиотици

има следните предимства:

а) висока бионаличност

б) широк спектър на антибактериално действие

в) бактерицидно действие

г) дълъг полуживот

д) бъбречен път на екскреция

079. Флуорохинолоните се различават от хинолоните по следните свойства:

а) широк антибактериален спектър на действие

б) бактериостатично действие

в) високо проникване в тъканта

г) пост-антибактериален ефект

д) орален път на приложение

080. Изберете лекарство,

максимално потискане на секрецията на солна киселина:

а) пирензепин

б) циметидин

в) карбеноксолон

г) антиациди

д) омепразол

081. Максималният брой странични ефекти сред H2-блокерите

обаждания:

а) циметидин

б) роксатидин

в) низатидин

г) ранитидин

д) фамотидин

082. Синтетични аналози на простагландини (енпростил, мизопростол)

произвеждат следните ефекти:

а) антисекреторно действие

б) секреция на барбитурати

в) образуване на слуз

г) репаративно действие

083. Инхибира метаболизма на други лекарства:

а) омепразол

б) карбеноксолон

в) циметидин

г) фамотидин

д) гастроцепин

084. Продължителността на антисекреторното действие на омепразол е:

а) 2-4 часа

б) 8-10 часа

в) 16-20 часа

д) 3 дни

085. Посочете лекарството,

който е мощен стимулант на образуването на слуз в стомаха:

а) карбеноксолон

б) платифилин

г) омепразол

д) метоклопрамид

086. При наличие на бъбречна недостатъчност

необходимо е коригиране на дозата:

а) аналози на простагландин

б) омепразол

в) H2 блокери

г) сукралфат

д) антихолинергици

087. Посочете антисекреторното лекарство, блокиращо "протонната помпа"

а) метоклопрамид

б) карбеноксолон

в) пирензепин

г) сукралфат

д) омепразол

088. При бъбречна патология

настъпват следните промени във фармакогенетиката на лекарствата:

а) нарушена бъбречна екскреция

б) повишаване на концентрацията на лекарства в кръвната плазма

в) намалено свързване с плазмените протеини

г) увеличаване на полуживота

д) намаляване на бионаличността

089. Цирозата на черния дроб причинява следните промени във фармакокинетиката на лекарствата:

а) намаляване на метаболизма при първо преминаване

б) намалено свързване с плазмените протеини

в) увеличаване на полуживота

г) повишаване на бионаличността

д) намаляване на обема на разпределение

090. При сърдечна недостатъчност

се наблюдават следните промени във фармакокинетиката на дигоксин:

а) намаляване на абсорбцията в стомашно-чревния тракт с 30%

б) намалено свързване с плазмените протеини

в) повишен метаболизъм в черния дроб

г) намалена бъбречна екскреция

д) увеличаване на полуживота

091. Алкохолът причинява:

а) за увеличаване на абсорбцията на лекарства

в) за забавяне на метаболизма в черния дроб

г) намаляване на бъбречната екскреция

д) до увеличаване на полуживота

092. Никотинът води до:

а) за намаляване на абсорбцията на лекарства

б) да се увеличи обемът на разпространение на лекарствата

в) за увеличаване на връзката с плазмените протеини

г) за ускоряване на метаболизма в черния дроб

д) за увеличаване на бъбречната екскреция на лекарства

093. При ангина пекторис е показано:

а) нифедипин

б) пропранолол

в) капотен

г) еналаприл

д) клонидин

094. При ангина на Принцметал (вазоспастична) е показано:

а) нифедипин

б) обзидан

в) дипиридамол

г) допегит

д) каптоприл

095. Критерият за ефективност на антиангинозно средство е:

а) увеличаване на времето за зареждане на PEM > 1 минута

б) увеличаване на количеството консумирана NTG

в) Увеличаване на времето на VEM - проби >2 минути

г) намаляване на времето за зареждане

д) преходът на пациента от 2-ри към 3-ти функционален клас на ангина пекторис

096. Антиангиналните лекарства включват:

а) камбанки

б) капотен

в) аспирин

г) верапамил

097. За спиране на пристъп на ангина пекторис се използват:

а) става

б) нитронг

в) нитроглицерин

г) верапамил

д) дилтиазем

098. От антиангинални лекарства

с комбинация от коронарна артериална болест и артериална хипертония, е показано:

а) става

б) верапамил

в) каптоприлил

г) камбанки

д) еналаприл

099. Развитието на толерантност е най-вероятно при използване на:

а) тринитролонга

б) сустака

в) сублингвален нитроглицерин

г) изосорбитол-5-мононитрат

д) нитронг

100. Методът за проследяване на ефективността на антиангинозната терапия е:

а) Холтер ЕКГ мониторинг

б) контрол на нивата на кръвните липиди

в) ежедневно мониториране на артериалното налягане

г) измерване на дихателната функция (функции на външното дишане)

д) измерване на артериалното налягане при орто- и клиностаза

101. Лекарството на избор при ангина пекторис при пациент с брадикардия е:

а) пиндолол

б) пропранолол

в) верапамил

г) дилтиазем

д) метопролол

102. Лекарство на избор при ангина пекторис

при пациент със сърдечна недостатъчност е:

а) верапамил

б) коринфар

в) дилтиазем

г) ацебуталол

д) нитросорбитол

103. Инхалаторните кортикостероиди включват:

а) хидрокортизон

б) беклометазон

в) преднизон

г) полкортолон

д) дексаметазон

104. Дългодействащите селективни 2-агонисти включват:

а) флутиказон

б) салметерол

в) салбутамол

г) фенатерол

д) тербуталин

105. За спиране на пристъп на бронхиална астма се използва:

а) ипратропиев бромид

б) теопек

г) салбутамол

д) задитен

106. Мембранният стабилизатор за перорално приложение е:

а) кетотифен

б) недокрамил натрий

в) натриев хромогликат

г) супрастин

д) ипратропиев бромид

107. При синдрома на "заключване" се прилагат:

а) салбутамол

б) фенотерол

в) теопек

г) еуфилин

д) адреналин

108. Муколитичните лекарства включват:

а) кодеин

б) натриев хромогликат

в) ацетилцистеин

г) салметерол

д) теофилин

109. При едновременна употреба

повишава концентрацията на теофилин в кръвта:

а) офлоксацин

б) пеницилини

в) цефтриаксон

г) гентамицин

д) бисептол

110. При едновременна употреба

намалява концентрацията на теофилин в кръвта:

а) пефлоксацин

б) циметидин

в) рифампицин

г) еритромицин

д) ампиокс

111. При повишено налягане в белодробната артерия

при пациент с бронхиална астма:

а) верапамил

б) нифедипин

в) дигоксин

д) беклометазон

112. При бронхиална астма на фона на хроничен бронхит е показано:

а) ипратропиев бромид

б) адреналин

в) ефедрин

г) кетотифен

д) супрастин

113. Посочете предпочитания начин на приложение на лекарството

при застойна сърдечна недостатъчност:

а) ректално

б) сублингвални

в) вътре

г) венозно

д) кожа

114. Избройте лекарства,

имащи директен положителен инотропен ефект:

а) дигоксин

б) допамин

в) норепинефрин

г) еуфилин

д) хидралазин

115. Посочете състоянията,

повишена чувствителност към сърдечни гликозиди:

а) напреднала възраст

б) тиреотоксикоза

в) белодробно сърце

г) хипокалиемия

д) застойна сърдечна недостатъчност

116. Избройте лекарствата, при взаимодействие с които

Концентрацията на дигоксин в кръвта може да се увеличи:

а) фосфолагел

б) хинидин

в) верапамил

г) амиодарон

117. Избройте факторите, забавящи абсорбцията на сърдечните гликозиди

от стомашно-чревния тракт:

а) хронична бъбречна недостатъчност

б) застойна сърдечна недостатъчност

в) стомашна язва

г) едновременно приложение с антиациди

118. Избройте факторите

осигуряване на най-голяма безопасност и ефективност на диуретиците

при продължително лечение на сърдечна недостатъчност:

а) максимални дози

б) средни дози

в) минимални дози

г) дневен прием

д) периодично приемане

119. Посочете най-ефективния диуретик

за лечение на застойна сърдечна недостатъчност

с развитието на вторичен хипералдостеронизъм:

а) етакринова киселина (урегид)

б) хлорталидон (хигротон)

в) ацетазоламид (диакарб)

г) спиронолактон (верошпирон)

д) триампур

120. Посочете основния терапевтичен ефект на нитросорбид

при пациенти със сърдечна недостатъчност:

а) разширяване предимно на артериолите и намаляване на следнатоварването

б) разширяване предимно на венули и намаляване на предварителното натоварване

в) директен положителен инотропен ефект

г) повишена диуреза и намалено преднатоварване

121. Избройте лекарства, които имат антихолинергични странични ефекти:

а) лидокаин

б) хинидин

в) амиодарон (кордарон)

г) верапамил

д) прокаинамид (новокаинамид)

122. Избройте групите антиаритмични лекарства,

имащи антифибрилиращ ефект:

а) сърдечни гликозиди

б) калциеви антагонисти (група 4)

в) блокери (група 2)

г) амиодарон, бретилиев тозилат (група 3)

д) хинидин, прокаинамид и други лекарства от група 1а

123. Избройте лекарства,

което може да провокира пристъп на предсърдна тахиаритмия

със синдром на Волф-Паркинсон-Уайт:

а) дигоксин

б) амиодарон

в) верапамил

г) пропранолол

д) етмозин

124. Посочете лекарства, които се характеризират с аритмогенен ефект:

а) аймалин

б) мексилетин

в) пропафенон

г) амиодарон

125. Избройте показанията за лечение с антиаритмични лекарства:

а) аритмии, причиняващи хемодинамични нарушения

б) субективна непоносимост към аритмия

в) нарушение на ритъма на високи градации

г) чести ритъмни нарушения

126. Избройте лекарства,

основното антиаритмично действие на които

свързани с удължаване на атриовентрикуларната проводимост:

а) пропранолол

б) лидокаин

в) верапамил

г) дигоксин

д) прокаинамид

127. Посочете ефектите от взаимодействието на амиодарон и дизопирамиди

(ритмилен, норпас):

а) забавяне на метаболизма на дизопирамид

б) забавяне на метаболизма на амиодарон

в) повишен риск от странични ефекти на дизопирамид

г) повишен риск от странични ефекти на амиодарон

128. Посочете страничния ефект на нифидепин:

а) брадикардия

б) бронхоспазъм

в) подуване на краката и стъпалата

г) развитие на F-V блокада

д) улцерогенност

129. Посочете лекарството,

механизмът на неговото хипотензивно действие

е блокада на -рецептори:

а) клонидин

б) празозин

в) пропранолол

г) капотен

д) верапамил

130. Лекарството по избор

при пациент с хипертония и застойна циркулаторна недостатъчност

е:

а) еналаприл

б) нитросорбид

в) клонидин

г) аделфан

д) пентамин

131. Оптималното лекарство за дългосрочна антихипертензивна терапия трябва да:

а) засягат метаболизма

б) имат реакции на рикошет

в) имате синдром на отнемане

г) имат стабилна концентрация в кръвта

д) предизвикват ортостатични реакции

132. Избройте групите антихипертензивни лекарства,

намаляване на активността на системата ренин-ангиотензин-алдостерон:

а) АСЕ инхибитори

б) блокери

в) централни агонисти

г) тиазидни диуретици

д) калциеви антагонисти

133. Посочете антихипертензивни лекарства,

които трябва да се използват с повишено внимание

с комбинация от захарен диабет и хипертония:

а) верапамил

б) пропранолол

в) дилтиазем

г) хипотиазид

д) еналаприл

134. Осъществяване на лекарствен мониторинг

необходими при лечението на следните групи лекарства:

а) антиконвулсант

б) 2-симпатикомиметици

в) метилксантини

г) глюкокортикоиди

д) М-холиномиметици

135. Развитието на асистолия е възможно при комбинация с пропранолол:

а) с фенобарбитал

б) с фуроземид

в) с верапамил

г) с фенитин

д) с циметидин

136. Рискът от токсични ефекти се увеличава при комбинацията на гентамицин:

а) с фуроземид

б) с пеницилин

в) с метилксантини

г) с макролиди

д) с глюкокортикоиди

КЛИНИЧНА ФАРМАКОЛОГИЯ

114 - a, b, c, d, e

115 - a, b, c, d, e

088 - а, б, в, г

089 - а, б, в, г

090 - а, б, г, д

124 - a, b, c, d, e

078 - а, б, в, г

079 - а, в, г, д

083 - а, б, в, г

Изберете правилното твърдение: а) бионаличността е количеството лекарства, постъпващи в системното кръвообращение, изразено като процент от приетата доза, б) бионаличността се определя от степента на адсорбция на лекарството в стомашно-чревния тракт и тежестта на ефекта на първо преминава през черния дроб в) бионаличността се определя по формулата: F = AUC (в / m или вътре) / AUC (в / в) г) бионаличността на лекарствата, когато се прилага интрамускулно, се определя от степента на неговата абсорбция и биотрансформация в тялото.
Отговор: a B C

2.
Отговор: Атровент

3.

Отговор: а, г

4.

Отговор:

5.
Отговор:

6.

д) Ксилитол
Отговор: а, в

7.

Отговор: а,б,г

8.
Отговор:

9. е приет с остър миокарден инфаркт, настъпил преди 5 часа Назначения: анаприлин 20 mg 4 пъти на ден перорално, хепарин интравенозно при 10 000 единици на всеки 4 часа В същото време е възможно да се постигне увеличаване на времето за съсирване на кръвта до 18-23 минути. На следващия ден пациентът е диагностициран с дясностранна пневмония на долния лоб.Бензилпеницилин натриева сол (1 000 000 IU на всеки 4 часа) е предписан интравенозно.След 4 часа времето за съсирване на кръвта е 8 минути. Каква е вашата тактика?
Отговор:

10.

11.
Отговор: Вит.В12 в доза 500 mcg/ден през ден, фолиева киселина в доза 1,5 mg/ден, железен сулфат (80 mg Fe2+) веднъж дневно

12.

Отговор: Вит.С

13.

Отговор: Церебролизин

14.
алергии (към бутадион, хепарин, метиндол, пеницилин, теофилин) В болницата се предписва реопирин 5 ml интрамускулно 1 път на ден, хидрокортизон хемисукцинат 100 mg в кухината на колянните стави, tavegil 0,001 g 2 пъти на ден. ден.След 3 дни b-noy разви сърбящи еритематозни обриви по кожата на тялото.Каква е най-вероятната причина
влошаване?
Отговор:

15.



Отговор: a,b,e,f,h,i

16.
Отговор: След няколко месеца

17.


Отговор: a, b, c, e, f

18.
Отговор: a, b, c, d, e, g, h

19.
Отговор:

20.
Отговор: Ципрофлоксацин

21. Феноменът на първото преминаване на лекарства през черния дроб зависи от: а) кръвоснабдяването на черния дроб, б) свързването на лекарството с протеина, в) активността на хепатоцитните ензими, г) нивото на екскреция на лекарството, д) скоростта на абсорбция
Отговор: а, в

22. Лекарства, повлияващи микрозомалните чернодробни ензими: индуктори на микрозомалните чернодробни ензими: а) пеницилин, б) нитроглицерин, в) фенобарбитал, г) фуроземид, д) бутадион, е) кортизол, ж) пропранолол, з) циметидин, i) левомицетин, k ) дифенин
Отговор: c,d

23. е приета в отделението с жена за болка в дясната млечна жлеза, повишаване на T. до 39,5 C. Тя се разболя преди 3 дни, на 10-ия ден след раждането. При постъпване в отделението в горен външен квадрант на дясна млечна жлеза се установява хиперемия на кожата, масивен инфилтрат с флуктуация в центъра.Диагноза: остър десен мастит. Взета е култура от рани. Определете антибиотик на първи избор.
Отговор: Цефазолин

24.

Отговор: Анафилактична реакция

25.

Отговор: Левомицетин

26.
Отговор:

27.
Отговор: бигуаниди

28.

Отговор: Хипотония, световъртеж.

29.

Оценете действията на лекаря.

30. Пациент Д., 53 г., с диагноза коронарна артериална болест, стабилна стенокардия??? FC, постинфарктна кардиосклероза, предсърдно мъждене, HNK??B чл. Приема строфантин, дигоксин, фуроземид, панангин в средни терапевтични дози. Неочаквано температурата на пациента се повиши до 38,4 ° C, появиха се кашлица, задух, крепитус в белите дробове вдясно. На рентгенограмата на белите дробове вдясно в долния лоб се определя зона на инфилтрация. Към лечението са добавени гентамицин, сулфокамфокаин, супрастин.

Отговор:

31.

Отговор: Фентоламин.

32.

Отговор: d,d

33.

Отговор:

34.
Отговор:

35.
Отговор: Еналаприл.

36.

Отговор: а,б,г

37.

Отговор:

38.
Отговор:

39.
Отговор:

40.

Отговор: Всички изброени

41. Лекарства, повлияващи микрозомалните чернодробни ензими: инхибитори на микрозомалните чернодробни ензими:
а) пеницилин, б) нитроглицерин, в) фенобарбитал,
г) фуроземид, д) бутадион, е) кортизол, ж) пропранолол,
h) циметидин, i) хлорамфеникол, j) дифенин
Отговор: h,i

42.
Отговор: След 7-14 дни

43. Посочете комбинация от лекарства, която води до конкуренция за свързване с протеини, което може да доведе до опасно повишаване на съдържанието на свободната фракция на едно от лекарствата в кръвта и появата на симптоми на предозиране:
Отговор: неодикумарин и бутадион

44. Изберете лекарства с тесен терапевтичен прозорец:
а) пеницилини, б) антиконвулсанти,
в) антиаритмични лекарства, г) дигоксин, д) метотрексат, е) теофилин, ж) циклоспорин, з) макролиди
Отговор: b, c, d, e, f, g

45. Посочете комбинации от лекарства, при които поради конкуренция за свързване с протеините се наблюдава повишаване на концентрацията на свободната фракция на един от тях в кръвната плазма: а.строфантин и мисклерон, б.дигитоксин и мисклерон, в.неодикумарин и бутадион. , г. нифедипин и хидрохлоротиазид
Отговор: b,c

46. Открити са чести камерни екстрасистоли и пароксизми на предсърдно мъждене HR 74 на минута, кръвно налягане 140/80 mm Hg.-nogo: а) хинидин,
б) Бонекор, в) Етацизин,

Отговор: а, б

47. Известно е, че при комбинация от хинидин и дигоксин често се наблюдава гликозидна интоксикация, с какво е свързана? Фармакодинамично взаимодействие:
Отговор: синергия

48. Известно е, че при комбинация от хинидин и дигоксин се наблюдава гликозидна интоксикация, с какво е свързана? Фармакокинетично взаимодействие, ефектът на хинидин върху:
Отговор: протеинова връзка

49. Критични периоди на вътрематочно развитие:
А. период на предимплантационно развитие (1 седмица)
b. етапът на ембриогенезата завършва до 8 седмици.
V. етапът на ембриогенезата завършва до 8 месеца.
периода непосредствено преди раждането
Отговор: а,б,г

50. Изберете от следните лекарства, които имат следните свойства: Антимикробни средства, чиято употреба е практически безопасна по време на бременност: а) сулфонамиди, включително бисептол,
б) аминогликозиди, тетрациклини, рифампицини, метронидазол (през 1 триместър на бременността), в. пеницилини, цефалоспорини, еритромицин, линкомицин, фузидин, г. антимикотични средства, антинеопластични средства
антибиотици.
Отговор: V

51. Метронидазол е предписан на кърмеща майка, посочете страничните ефекти:
а. повишена възбудимост, тахикардия, б. потискане на апетита, повръщане, в. потискане на ЦНС, дишане, загуба на тегло, г. повишена секреция на пролактин, набъбване на гърдите, е. надбъбречна хипоплазия, метаболитни нарушения, повишен риск от билирубинова енцефалопатия, е. кръвоизливи , дихателна недостатъчност, ацидоза, потискане на хемопоезата, анемия, недохранване, дисбиоза.
Отговор: b

52. Антимикробни средства на първи избор при новородени: а.бензилпеницилин, оксацилин, карбеницилин, гентамицин, амикацин, б.бензилпеницилин, оксацилин, бицилини, цефазолин, цефотаксим, еритромицин, линкомицин, нистатин, в. карбеницилин, гентамицин, сизомицин, амикацин, тобрамицин, цепорин (с неефективността на цефалоспорините от първо поколение), например еритромицин, линкомицин, нистатин, леворин, карбеницилин,
гентамицин, сизомицин
Отговор: b

53.
Отговор:

54. Основните характеристики на фармакокинетиката на лекарствата при възрастни хора:
а) намаляване на скоростта на абсорбция, б) ускоряване на абсорбцията, в) намаляване на скоростта на разпределение, г) ускоряване на разпределението, д. намаляване на свързването на лекарствата с плазмените протеини, д. повишаване на свързването на лекарствата към плазмените протеини, g. забавяне на метаболизма, h. ускоряване на метаболизма,
и забавяне на отделянето на лекарства, за ускоряване на отделянето на лекарства.
Отговор: a, c, e, g, i

55.
Отговор: b, c, d

56. Посочете страничните ефекти на бета-блокерите: а) брадикардия, б) артериална хипотония, в) бронхоспазъм, г) тахикардия, д) дисфункция на щитовидната жлеза,
е) интермитентно накуцване, ж) AV блокада
Отговор: a, b, c, f, g

57.
състояния: а) Естествен ход на заболяването, б) Развитие на толерантност към нитрати, в) Синдром на междукоронарна кражба, г) Поява на синдром на отскачане д) Явления на идиосинкразия
Отговор: а, б

58. Посочете страничните ефекти на амиодарон: а) брадикардия, б) артериална хипотония, в) бронхоспазъм, г) тахикардия, д) дисфункция на щитовидната жлеза, е) интермитентно накуцване, ж) AV блокада
Отговор: a, c, e, g

59. Как ще се промени вашата антиангинозна терапия, ако по време на нитратна терапия настъпи мозъчен инсулт?
Отговор: премахването на нитратите и назначаването на антиангинозно лекарство от друга група

60. Кои антихипертензивни лекарства се считат за най-безопасни за пациенти в напреднала възраст: а) бета-блокери, б) ганглийни блокери, в) симпатиколитици, г) бавни блокери на калциевите канали, д) тиазидни
диуретици, д) АСЕ инхибитори.
Отговор: d,d

61. Режим на лечение с Cordaron:
Отговор: по схемата, която включва постепенно намаляване на дозата от 600 mg до 200 mg на ден

62. Как инхибиторите на МАО (антидепресанти) влияят върху пресорния ефект на директните и индиректните адреностимуланти?
Отговор: засилване на ефекта

63. Небензодиазепинов бензодиазепинов рецепторен агонист:
Отговор: Золпидем

64. Хипнотичен - съединение от алифатната серия:
Отговор: Хлорал хидрат

65.

Отговор: А (б)

66. Протамин сулфат се предписва при предозиране:
Отговор: Хепарин

67. Какъв метод за детоксикация е най-ефективен при отравяне с вещества, които се свързват с кръвните протеини и липиди?
Отговор: Хемосорбция

68. Принципът на действие на налоксон при остро отравяне с морфин:
Отговор: Пречи на действието на морфина върху опиоидните рецептори

69. Посочете лекарствата с антиоксидантни свойства: а) верапамил б) вит.А, в) вит.К, г) вит.С, д) вит.Е, е) селен, ж) карнозин, з) доксициклин
Отговор: b, d, e, f, g

70. Какви са ефектите на невролептиците?
а) антипсихотик б) седатив в) антиеметик
Отговор: a B C

71. 64-годишен пациент разви остър пристъп на закритоъгълна глаукома със силна болка в дясното око, излъчваща се към главата. Появиха се гадене и повръщане, задух, признаци на хипертонична криза тип 2 със сърдечна честота 62.
при мин. АН 200/140 mmHg.Б-Ная от много години страда от хипертония.В белите дробове има голям брой влажни ситно мехурчета. А. Clopamid, b. Veroshpiron, c. Hypothiazid, g. Furosemide IV, D. Diakarb:
Отговор: d,d

72. страда от захарен диабет от 15 години, за който получава инсулин на 70 единици / ден, което поддържа нивото на гликемия в диапазона от 7,5-8,6 mmol / l. Напоследък кръвното налягане се повишава до
170/90-180/100 mm Hg, във връзка с което лекуващият лекар предписва обзидан в дневна доза от 120 mg Какви нежелани реакции трябва да се очакват при тази комбинация от лекарства? а. Хипергликемия до кома б. Сърдечна недостатъчност в. Хипогликемия до кома г. Ортостатична хипотония д. Хипертония
Отговор: b,c

73. , страдащи от хормонално-зависима бронхиална астма, предписаха преднизолон (5 mg дневно), салбутамол (вдишване на 2 дози аерозол 4 пъти на ден) Развива се б-та екзацербация на бронхиална астма Каква е причината за това?
А. Фенобарбитал ускори биотрансформацията на: а. салбутамол, б. преднизолон, Б. Фенобарбитал ускори екскрецията на: а. салбутамол, б. преднизолон, Б. Фенобарбитал забави екскрецията на: а. салбутамол, б. салбутамол, б. .преднизолон
Отговор: А (б)

74. страдащ от коронарна артериална болест, ангина пекторис III FC Сърдечна честота 90 в минута, кръвно налягане 150/80 mm Hg Анамнеза за хроничен бронхит с бронхоспастичен синдром в ремисия За антиангинална терапия. а) Нитрати и верапамил, б. Нитрати и атенолол
в) Нитрати и анаприлин, г) Нитрати и нифедипин,
д) Нифедипин и амиодарон
Отговор: А

75. за стенокардия приема нитросорбид 10 mg 4 пъти на ден.Сърдечна честота 80 в mi.BP 140 / 80 mm Hg.След 1 месец след началото на терапията пристъпите на ангина пекторис зачестиха.Какви са възможните причини за влошаване
състояния: а) Естествен ход на заболяването, б) Развитие на толерантност към нитрати, в) Синдром на междукоронарна кражба, г) Поява на синдром на отскок, д) явления на идиосинкразия
Отговор: а, б

76. пристъпи на ангина пекторис се отбелязват при умерено физическо натоварване.В анамнезата, колаптоидно състояние след еднократна доза нитроглицерин сублингвално (оттогава b-noy нитроглицерин не е приеман). Съпътстващи заболявания-GB (работно ниво на кръвното налягане 160/100 mm Hg.
Чл., хипофункция на щитовидната жлеза По време на прегледа кръвното налягане е 190/100 mm Hg, сърдечната честота е 72 в минута. Пациентът е противопоказан:
Отговор: Амиодарон

77. поради артериална хипертония от 2-ра степен получава 0,000075 g клонидин 4 пъти на ден.Във връзка с развитието на сенилна депресия е предписан мелипрамин.3 дни след назначаването на мелипрамин настъпва хипертонична криза.състояния: а) Последица от естествения ход на заболяването, б) Последица от хипертензивния ефект на мелипрамин, в) Последица от неблагоприятни лекарствени взаимодействия, г) Последица от възможно прекъсване на приема на лекарството и развитие на синдром на отнемане.
Отговор: b, c, d

78. поради хипертоничната криза се прилага интравенозно натриев нитропрусид в големи дози (със скорост 8 μg/min). Появиха се задух, акроцианоза, притискащи болки зад гръдната кост, мускулни потрепвания.На какво се дължи влошаването на б-та?
Отговор: Токсичен ефект на цианида

79. Установени са чести камерни екстрасистоли и пароксизми на предсърдно мъждене HR 74 в минута, кръвно налягане 140/80 mmHg.
за по-нататъшно лечение на b-nogo: а) хинидин, б) бонекор, в) етацизин,
г) мекситил, д) верапамил, е) пропранолол
Отговор: а, б

80. пароксизмална суправентрикуларна тахикардия на фона на синдром на WPW Aymalin е избран за спиране на атаката Определете оптималния режим на лечение с избраното лекарство: а) 1 mg / kg IV за повече от 10 минути, ако е необходимо, повторете след 30 минути, b ) 50 mg IV при изтичане-
3-5 минути в 10 ml 5% разтвор на глюкоза или изотоничен разтвор на NaCl или / m, в) 0,5-1 g / на всеки 2 минути се прилагат 0,1-0,2 g или / m
г) след парентерално приложение, предписвайте перорално 100 mg 4-5 пъти на ден, поддържаща доза от 50 mg 3-4 пъти на ден
Отговор: а, г

81. 28-годишен с диагноза SLE на фона на CRF разви оток на краката, увеличен черен дроб.Ехокардиографско изследване установи намаляване на сърдечния дебит. Сърдечна честота 95 в минута, кръвно налягане 170/100 mm Hg Какви сърдечни гликозиди са показани на пациента?
Отговор: Дигитоксин

82. 28-годишен с диагноза SLE на фона на CRF разви оток на краката, увеличен черен дроб.Ехокардиографията разкрива намаляване на сърдечния дебит.Сърдечна честота 95 на минута, кръвно налягане 170/100 mm Hg Пациентът приема дигитоксин.Във връзка с появата на конвулсивен синдром допълнително се предписва фенобарбитал (0,3 g / ден).
Отговор: След 7-14 дни

83. 57 години за постинфарктна атерокардиосклероза, застойна сърдечна недостатъчност от 2-ра степен получава 40 mg фуроземид IV и 300 mg
veroshpiron вътре Каква диуретична терапия ще предпише на b-nom в случай на рефрактерност?
Отговор: Фуроземид 80 mg IV и спиронолактон 300 mg перорално

84. страда от неатопична бронхиална астма, придружена от профузна бронхорея Пулс 62 мин. АН 140/80 mm Hg Кои лекарства са за предпочитане?
Отговор: Атровент

85. упорито рецидивиращ синдром на бронхиална обструкция с намалена чувствителност към холинергични и адренотропни лекарства Бронхиална астма страда от повече от 10 години Какво може да се предпише за намаляване на честотата и тежестта на астматичните пристъпи: а) Бета инхалация
2-адренергични стимуланти повече от 6 пъти на ден, б) Инхалация на m-антихолинергичен блокер, в) Прилагане на адреналин п/к в по-висока доза от обичайната за облекчаване на бронхоспазма, г) Еуфилин IV, д) Инхалаторни глюкокортикоиди.
Отговор: d,d

86. е приет за киселини, болка в епигастралната област на празен стомах, облекчена от натриев бикарбонат FEGDS разкрива язва (0,5 cm в диаметър) в 12 p.
zuyuschaya функция със средна интензивност с ниски алкални резерви, холинергичен тип рецепция. Диагноза: пептична язва 12 бр в остър стадий. Изберете най-ефективното и безопасно лекарство и определете неговия режим на дозиране:
Отговор: Пирензепин преди хранене 0,05 g 3 пъти на ден в продължение на 2 дни, след това 0,05 g 2 пъти на ден

87. открита дискинезия на хипертоничен тип на жлъчния мехур. Изберете най-добрия вариант за лечение.
Отговор: No-shpa 1-2 таблетки 3 пъти на ден, отвара от безсмъртниче 1/2 чаша 30 минути преди хранене

88. страда от хроничен холецистопанкреатит в продължение на 5 години.През последната седмица след нарушение на диетата се наблюдава засилване на болката в дясната гръдна клетка, гадене, горчивина в устата.Изберете най-ефективните холеретични средства, които едновременно имат антимикробна активност:
а) Алохол, б) Холензим, в) Никодин, г) Отвара от вратига, д) Ксилитол
Отговор: а, в

89. с намерение за самоубийство изпила 20 таблетки феназепам.2 часа след приема на лекарството била откарана в болница. B-I е в съзнание, но рязко потиснат.Извършена е стомашна промивка.Изберете най-оптималните лаксативи: а) глауберова сол, б) магнезиев сулфат, в) екстракт от кора на зърнастец, г) бисакодил,
д) рициново масло, е) морски водорасли, ж) вазелиново масло
Отговор: а,б,г

90. Мъж на 46 години е приет в кардиоинтензивно отделение с остър трансмурален инфаркт на миокарда, настъпил преди около 5 часа.Назначения: анаприлин 20 mg 4 пъти дневно перорално, хепарин интравенозно по 10 000 единици на всеки 4 часа.В същото време време, беше възможно да се постигне увеличаване на времето за съсирване на кръвта до 18-23 минути. На 4-ия ден пациентът има микрохематурия (22 еритроцита в зрителното поле). Каква е вашата тактика?
Отговор: Намалете дозата на хепарина, докато времето за съсирване стане поне 10-12 минути

91. е приет с остър миокарден инфаркт, настъпил преди 5 часа Назначения: анаприлин 20 mg 4 пъти на ден перорално, хепарин интравенозно при 10 000 единици на всеки 4 часа В същото време е възможно да се постигне увеличаване на времето за съсирване на кръвта до 18-23 минути. На следващия ден пациентът е диагностициран с дясностранна пневмония на долния лоб.Бензилпеницилин натриева сол (1 000 000 IU на всеки 4 часа) е предписан интравенозно.След 4 часа времето за съсирване на кръвта е 8 минути. Каква е вашата тактика?
Отговор: Променете начина на приложение на пеницилин

92. претърпя радикална операция от рак на стомаха. На 4-ия ден след операцията изследването на коагулограмата разкрива хиперкоагулация и намаляване на фибринолитичната активност на кръвта.Препоръчително ли е да се предписват антикоагуланти?
Отговор: Показани са антикоагуланти, но е необходимо внимателно наблюдение за предотвратяване на хеморагичен синдром.

93. постъпил в болницата с оплакване от силна слабост, задух при ходене. По време на прегледа се установява анемия (хемоглобин-56 g/l) в кръвния тест, цветният индекс е 1,2, при изследване на езика - глосит.Пунктът на костния мозък разкрива мегалобластичен тип хемопоеза.Концентрацията на желязо в кръвният серум е в границите на нормата Диагноза: В 12 -дефицитна анамия Изберете най-оптималния вариант на лечение.
Отговор: Вит.В12 в доза от 500 mcg / ден през ден, фолиева киселина в доза от 1,5 mg / ден, железен сулфат (80 mg Fe2 +) 1 път на ден

94. след хипотермия се появиха студени тръпки, повишаване на телесната температура до 38,6 ° C, кашлица с мукопурулентна храчка, болка в дясната половина на гръдния кош.Клинично и рентгенологично е установена диагноза дясностранна пневмония на долния лоб.Назначено е лечение: цефазолин, 0,5 g 2 пъти на ден IM, hemodez 400 ml IV капково, отхрачваща смес от 1 супена лъжица 6 пъти на ден Изберете антиоксидантно лекарство, най-много
ефективно повлияване на процесите на свободнорадикално окисление в белите дробове, което трябва да се добави към провежданата терапия
Отговор: Вит.С

95. поради остър мозъчно-съдов инцидент от исхемичен тип преди 12 часа получава реополиглюкин 400 ml интравенозно капково
1 път на ден Изберете най-ефективното лекарство в тази ситуация с антиоксидантни свойства.
Отговор: Церебролизин

96. от 5 години страда от деформираща артроза на долните крайници с тежък синовит. Има история на употреба на наркотици
алергии (към бутадион, хепарин, метиндол, пеницилин, теофилин) В болницата се предписва реопирин 5 ml интрамускулно 1 път на ден, хидрокортизон хемисукцинат 100 mg в кухината на колянните стави, tavegil 0,001 g 2 пъти на ден. ден.След 3 дни b-noi разви сърбящи еритематозни обриви по кожата на тялото.Каква е най-вероятната причина за влошаването?
Отговор: лекарствена алергична реакция

97. диагнозата ревматоиден артрит е потвърдена Какви основни лекарства за лечение на ревматоиден артрит можете да предпишете: а) 4,7-хлорохинолонови лекарства (делагил), б) цитостатици (азатиоприн, циклофосфамид и др.), в) глюкокортикоиди (преднизолон). ), г) НСПВС,
д) златни препарати (кризанол), е) салазопиридазин,
ж) антибиотици (тетрациклини), з) D-пенициламин,
и) Имуномодулатори (левамизол)
Отговор: a,b,e,f,h,i

98. На пациент с ревматоиден артрит е предписан метотрексат. След колко време ще се прояви ефектът от метотрексата?
Отговор: След няколко месеца

99. с ревматоиден артрит е предписан метотрексат. Какви мерки ще предприемете, за да наблюдавате безопасността на фармакотерапията с метотрексат при този пациент: а) Пълна седмична кръвна картина
(за предпочитане 2 пъти седмично), б) Извършване на кръвен тест за определяне на броя на тромбоцитите на всеки 3-4 седмици,
в) Извършване на общ анализ на урината, г) Определяне на съдържанието на пикочна киселина, д) Провеждане на тест за скрита кръв в изпражненията, е) Определяне на съдържанието на трансаминази, общ билирубин на всеки 6-8 седмици
Отговор: a, b, c, e, f

100. за ревматизъм получава делагил продължително време Какви мерки ще предприемете за контрол на безопасността на терапията с делагил при продължително приложение: а) Пълна кръвна картина, б) Пълен анализ на урината, в) ЕКГ, г) Изследване на очните дъна, д) Изследване на зрителното поле, а) Рентгеново изследване на гръдния кош, ж) Определяне на броя на тромбоцитите, з) Изследване на роговицата
Отговор: a, b, c, d, e, g, h

101. 39 г., ревматоиден артрит, предимно ставна форма, 2 степен на активност Какви варианти на комбинирана терапия биха били подходящи за този пациент?
Отговор: Delagil 0,25 g 3 пъти на ден, преднизолон 15 mg / ден, krizanol интрамускулно 1 ml 5% разтвор 1 път седмично

102. 63 г. страда от захарен диабет, приема глибенкламид.Постъпва в отделението с картина на остра дясно-долнолобна пневмония, потвърдена рентгеново.Предписан е хлорамфеникол, към който пациентката е имала алергична реакция.-noy разкрива ниско ниво на креатининов клирънс (24 ml / min), в резултат на което цефтриаксон е отменен Кое лекарство трябва да продължи лечението?
Отговор: Ципрофлоксацин

103. е приета в отделението с жена за болка в дясната млечна жлеза, повишаване на T. до 39,5 C. Тя се разболя преди 3 дни, на 10-ия ден след раждането. При постъпване в отделението в горен външен квадрант на дясна млечна жлеза
хиперемия на кожата, масивен инфилтрат с флуктуация в центъра.Диагноза: остър десностранен мастит.Б-ная е оперирана. Взета е култура от рани. Определете антибиотик на първи избор.
Отговор: Цефазолин

104. постъпи в отделението с картина на остър десен мастит.Заболя преди 3 дни на 10 ден след раждането. Б-ная оперирана.
Предписан е цефазолин.След 2-рата инжекция на лекарството, след 20 минути, се появи понижаване на кръвното налягане, замаяност, гадене, повръщане, неволно уриниране и конвулсивен синдром. Какво усложнение се е развило при пациента?
Отговор: Анафилактична реакция

105. 21 г. постъпи в отделението с картина на остър десен мастит.Заболя преди 3 дни на 10-тия ден след раждането.откроени рани от отделяне
пеницилиназа-образуващи staphylococcus aureus и Haemophilus influenzae , Изберете антибактериално лекарство, като вземете предвид бактериалната микрофлора и фармакокинетичните характеристики
Отговор: Левомицетин

106. страда от хроничен тонзилит и хроничен холецистит.При изследването в посявка от фаринкс и в посявка на жлъчка се установява Staphylococcus aureus, който образува пеницилиназа.В анамнезата се отбелязва алергия към оксацилин. Трябва ли да коригирам режима на дозиране на лекарството? Ако да, как?
Отговор: Намалете честотата на приложение и намалете дозата

107. 50 години се оплакват от обща слабост, жажда, често уриниране, сърбеж по кожата и външните гениталии.Прегледът разкрива затлъстяване (телесно тегло 96 kg при височина 168 cm).Кръвна захар 9,9 mmol/l, урина 1%, реакция към ацетон е отрицателен. Какви хипогликемични лекарства са оптимални в този случай?
Отговор: бигуаниди

108. 48 г., постъпва с оплаквания от притискащи болки, появяващи се при физическо натоварване, които се облекчават от нитроглицерин. Преди 3 години претърпя миокарден инфаркт. Везикуларно дишане в белите дробове. Сърдечните звуци са приглушени, систоличен шум на върха, чести екстрасистоли. Пулс - 92 в минута. BP - 100/60 mm Hg. Изкуство. Черният дроб не е увеличен, няма отоци. ЕКГ - синусова тахикардия, цикатрициални промени в миокарда, чести камерни екстрасистоли. Предписани са Obzidan 160 mg / ден, Sustak-Forte 19,2 mg / ден, Panangin, Riboksin.
Какъв страничен ефект е вероятен при пациент с тази комбинация от лекарства?
Отговор: Хипотония, световъртеж.

109. Пациент М. на 52 г. постъпва с оплаквания от задух, сърцебиене, болки в дясното подребрие, отоци по краката. От 18 години е на диспансерно лечение с диагноза ревматизъм. Кожата е бледа, акроцианоза, руж по бузите. В базалните части на белите дробове има недоловими хрипове с фини мехурчета. Границите на относителната тъпота на средата са разширени нагоре и надясно. Сърдечните звуци са приглушени, аритмични, систоличен шум на върха, акцент?? тон на белодробната артерия. Пулс-96 в мин. Пулс-140 в минута. BP - 130/85 mm Hg. Изкуство. Коремът е мек, черният дроб излиза на 3-4 см от ръба на ребрената дъга. Оток на краката. Дневна диуреза -650 мл. ЕКГ: няма P вълна, има F-F вълни, ритъмът е грешен. След интравенозно приложение на 10 ml 10% разтвор на новокаинамид: синусовият ритъм се възстановява със сърдечна честота 72 на минута, на пациента се предписва перорално новокаинамид 0,5 g 4 пъти на ден, дигоксин 0,25 mg 1 табл.
3 пъти на ден, фуроземид 40 mg перорално в продължение на 3 дни. След 5 дни пациентът разви гадене, повръщане, диария, световъртеж. ЕКГ: синусов ритъм, HR-76 в минута, PQ -0.20 s, QRS-0.1 s. Лекуващият лекар отменя дигоксин и фуроземид и предписва унитиол и калиеви препарати.
Оценете действията на лекаря.
Отговор: Действията на лекаря са правилни, тъй като не само се надвишава средната дневна доза дигоксин, но има и взаимодействие с прокаинамид за свързване с протеини.

110. Пациент Д., 53 г., диагностициран с коронарна болест на сърцето, стабилна стенокардия III FC, постинфарктна кардиосклероза, предсърдно мъждене, HNK??B чл. Приема строфантин, дигоксин, фуроземид, панангин в средни терапевтични дози. Неочаквано температурата на пациента се повиши до 38,4 ° C, появиха се кашлица, задух, крепитус в белите дробове вдясно. На рентгенограмата на белите дробове вдясно в долния лоб се определя зона на инфилтрация. Към лечението са добавени гентамицин, сулфокамфокаин, супрастин.
Какви странични ефекти от лечението е най-вероятно да се появят при пациент с такава комплексна терапия?
Отговор: Когато се комбинира с фуроземид, нефротоксичният ефект на гентамицин е най-вероятен.

111. Пациент на 28 години е приет с оплаквания от сърцебиене, главоболие, втрисане. По време на кризата, която се развива 2-4 пъти годишно, кръвното налягане се повишава до 260/110 mm Hg. Чл., сърдечна честота-140 в минута, бледа кожа, парещи болки в областта на сърцето, пулсация в главата, понякога повишаване на телесната температура до 38. След пристъпи на полиурия. В междупристъпния период кръвното налягане е 120/80 mm Hg. Изкуство. Обективно изследване на органична патология от вътрешните органи не е открито. При изследвания на кръв и урина без патология.
Посочете най-ефективното лекарство (първи ред) за облекчаване на криза при пациент:
Отговор: Фентоламин.

112. ревматоиден артрит по време на лечението с метотрексат се появи изразено кървене от носа Какво може да бъде причинено от: а) Увреждане на носните съдове поради подлежащия патологичен процес, б) Повишен PI поради токсичен хепатит, причинен от метотрексат, в) Повишен тромбоцитна агрегация под въздействието на метотрексат,
г) Индуцирано от лекарства намаляване на броя на тромбоцитите, д) Токсичен ефект на метотрексат върху назалните съдове
Отговор: d,d

113. Пациент К., 62 години, артериална хипертония 1-ва степен. Последното влошаване на състоянието се дължи на психо-емоционален стрес. При преглед: състоянието е относително задоволително, слабо главоболие. AD-170/100 mm Hg ("работно" BP-120/70 mm Hg), сърдечна честота-90 в минута. Лекуващият лекар предписва анаприлин 60 mg / ден, верапамил 160 mg / ден.
Какви промени можете да очаквате, когато предписвате верапамил в допълнение към анаприлин?
Отговор: Засилване на отрицателния дромотропен ефект.

114. Пациент С., 56 години, приема нитросорбид (10 mg) 1m x 4r на ден за ангина пекторис. Как ще се промени тактиката на антиангинозната терапия, ако пациентът развие церебрален инсулт по време на лечение с нитрати?
Отговор: Отменете нитратите и предпишете антиангинозно лекарство от друга група.

115. Пациент на 42 години с хроничен гломерулонефрит и артериална хипертония. При постъпване: АН 200/120 mm Hg, пулс 75-80 удара в минута, отоци по лицето, кръста, краката. Общ протеин в кръвния серум 3,8 g%, протеин в урината 16 g / l Посочете най-ефективните лекарства за антихипертензивна терапия при този пациент:
Отговор: Еналаприл.

116. страдащи от постоянно рецидивиращ синдром на бронхиална обструкция, лекарят инжектира s / c 1 ml адреналин Какви са проявите на токсични
Действията на адреналина са възможни в тази ситуация: а) възбуждане на ЦНС, б) екстрасистолия, в) токсично увреждане на черния дроб, г) тахикардия, д) блокада на импулсната проводимост по проводната система на сърцето.
Отговор: а,б,г

117. 57 години за слединфарктна артиосклероза, застойна сърдечна недостатъчност 2В степен получава 40 mg фуроземид IV и 300 mg
veroshpiron вътре Каква диуретична терапия ще предпише на b-nom в случай на рефрактерност?
Отговор: Фуроземид 80 mg IV и спиронолактон 300 mg перорално

118. страда от неатопична бронхиална астма, съпроводена с профузна бронхорея Пулс 62 мин. АН 140/80 mm Hg. След назначаването на атропин сулфат b-noi първо отбеляза подобрение в състоянието - бронхореята рязко намаля, но 10 дни след началото на лечението състоянието отново се влоши: треска (37,8 ° С), задух, кашлица с храчки трудно се отделя, появи се пулс 90 в минута. Какви са причините за такива промени в състоянието на пациента?
Отговор: Нарушаване на отделянето на храчки с последваща инфекция

119. Жена на 52 години страда от хипертония?? Изкуство. Приема резерпин 1 табл. (0,0001) 3 пъти на ден. BP се нормализира след 1 седмица. След 4 седмици редовен прием се появиха "гладни" болки в епигастричния регион, по време на гастроскопия беше диагностициран ерозивен дуоденит. Как си обяснявате появата му?
Отговор: Повишаване на тонуса на n vagus на фона на резерпин и повишаване на стомашната секреция.

120. 60-годишен пациент с коронарна болест на сърцето, стабилна ангина пекторис IV f.c. Предписан е Cordarone 600 mg / ден (като антиангинозно лекарство).
Какви нежелани реакции могат да се наблюдават при пациент с продължителна употреба на кордарон?
Отговор: Всички изброени

121. При избора на режим на дозиране на лекарството въз основа на T?
определи:
Отговор: честота на приемане

122. По-точно характеризира скоростта на отделяне на лекарства от тялото:
Отговор: общ просвет

123. Връзката на лекарствата с плазмените протеини:
Отговор: определя възможността за развитие на странични ефекти при комбиниране на лекарства

124. Важно е да се определи стойността на бионаличността:
Отговор: пътища на приложение на лекарства*

125. При продължителна употреба на силни диуретици може да изпитате:
Отговор: нарушен глюкозен толеранс

126. Замаяност, липса на усещане в крайниците, затруднено влизане и излизане без визуален контрол и други симптоми на токсични ефекти се появяват при 75% от пациентите, които:
Отговор: получават стрептомицин

127. Предозирането на симпатикомиметици причинява:
Отговор: ритъмни нарушения

128. Нежеланите реакции, свързани с антибиотика моксалактам, включват следното:
Отговор: тромбоцитопения

129. Комбинираната употреба на индометацин и гентамицин най-често причинява:
Отговор: бъбречна дисфункция

130. Използването на клавуланова киселина в комбинация с амоксицилин позволява:
Отговор: разширяване на спектъра на действие на амоксицилин върху щамове бактерии, които произвеждат бета-лактамаза

131. Предвижда се допълнително да се предпише друго антиаритмично лекарство от клас 1 на пациент, получаващ дифенин за дълго време.Кое антиаритмично лекарство ще изисква увеличение на дозата с 20-30% от стандартната?
Отговор: всички лекарства

132. Едновременното поглъщане на тетрациклин и Ca2 + препарати ще допринесе за:
Отговор: намалена абсорбция на тетрациклин

133. В същото време назначаването на хлорамфеникол и аценокумарол може да доведе до:
Отговор: за намаляване на антибактериалната активност на хлорамфеникола

134. За сърдечна недостатъчност:
Отговор: допаминът причинява вазоконстрикция на бъбречната кора във високи дози (повече от 10 mcg / kg / min)

135. Артериалната хипертония се характеризира с:
Отговор: повишаване на концентрацията на натрий в съдовата стена

136. Апресин (хидралазин):
Отговор: причинява тахикардия

137. Бета-блокерите причиняват:
Отговор: намаляване на сърдечната честота

138. Следните твърдения за алфа-блокерите са верни:
Отговор: всичко е точно

139. Бета-1 - блокери:
Отговор: селективно действат върху бета1-адренергичните рецептори, лекарствата са безопасни за бронхиална астма

140. Показания за употребата на бета-блокери са:
Отговор: сърдечни аритмии

141. Посочете верните твърдения:
Отговор: строфантинът се разрушава до голяма степен в стомашно-чревния тракт и следователно приемането му е нерационално

142. Индикации за назначаване на SG:
Отговор: CNC при пациенти с исхемична болест на сърцето, постинфарктна кардиосклероза и постоянна форма на предсърдна тахиаритмия

143. Фактор, който повишава риска от развитие на SG интоксикация:
Отговор: хипокалиемия

144. Състояние, което повишава риска от развитие на интоксикация с FH:
Отговор: хипотиреоидизъм

145. За да намалите риска от развитие на толерантност към нитрати, трябва:
Отговор: правете паузи между лекарствата

146. В случай на развитие на толерантност към sustac, той може да бъде заменен с:
Отговор: корватон

147. Главоболието може да бъде причинено от:
Отговор: Отговорите A, B, C са верни

148. Механизмът на действие е подобен на нитроглицерина:
Отговор: молсидомин

149. Предозирането на кои лекарства може да причини ортостатична хипотония?
Отговор: нитрати

150. Назовете група антиаритмици, които увеличават продължителността на потенциала на действие:
Отговор: блокери на калиеви канали

151. Кое от следните лекарства има най-силно изразен отрицателен инотропен ефект?
Отговор: дизопирамид

152. Кои от несърдечните странични ефекти са характерни за повечето лекарства от клас 1C?
Отговор: зрително увреждане

153. По време на лечение с дизопирамид може да се влоши следното заболяване:
Отговор: доброкачествена хиперплазия на простатата с уринарна инконтиненция

154. Кога трябва да се промени дозата на лидокаин от стандартната доза?
Отговор: при пациенти с чернодробна недостатъчност

155. Посочете методите за наблюдение на ефективността на диуретиците при едематозен синдром:
Отговор: всичко е точно

156. Посочете методите за контрол върху безопасността на диуретиците при едематозен синдром:
Отговор: всичко е точно

157. Посочете ефективен и безопасен начин за попълване на запасите от калий в организма:
Отговор: назначаването на панангин вътре в 2 таблетки 3 пъти на ден

158. Посочете рисковите фактори за страничните ефекти на "бримковите" диуретици:
Отговор: дневна диуреза повече от 3 литра след въвеждането на диуретик

159. Посочете началото на действие на спиронолактон:
Отговор: 4-5 дни

160. За алкализиране на урината се използват следните методи, с изключение на:
Отговор: калиев цитрат 3 mg на всеки 6 часа

161. Урината може да бъде кисела с някое от следните лекарства, с изключение на:
Отговор: метионин

162. Посочете грешните позиции:
Отговор: без грешни позиции

163. Разпределете тези лекарства според степента на натрупване:
Отговор: неодикумарин

164. Изберете твърденията, които са напълно верни за препарата стрептокиназа:
Отговор: всичко е точно

165. Изберете фактор, който причинява тромбоза или допринася за образуването на тромби:
Отговор: всичко е точно

166. Кое от следните лекарства може да намали ефекта на индиректните антикоагуланти?
Отговор: рифампицин

167. Какви нежелани реакции могат да възникнат при употребата на хепарин?
Отговор: Всички изброени

168. На пациент с бронхиална астма, който дълго време е приемал теофилини с продължително действие, се предписва ципрофлоксация поради развитието на инфекция на пикочните пътища. В този случай е необходимо:
Отговор: намалете дозата на теофилин с 30%

169. Дете, получаващо карбамазепин дълго време поради наличието на епилепсия, развива бронхообструктивен синдром с дихателна недостатъчност на 2-ра супена лъжица. Когато се предписва аминофилин на такъв пациент:
Отговор: дозата на аминофилин трябва да се увеличи с 1,5 пъти

170. Когато предписвате теофилин на пушач:
Отговор: дозата трябва да се увеличи

171. Посочете лекарството, което намалява елиминирането на теофилин при едновременно приложение:
Отговор: циметидин

172. При пациент с бронхиална астма, който дълго време е приемал теотард, се появяват гадене, повръщане, главоболие и безсъние на фона на грипна инфекция и треска. Менингеалните симптоми са отрицателни. Терапевтична тактика в този случай:
Отговор: спрете теотард или намалете дозата му с 50%

173. Страничните ефекти на теофилин могат да включват следното с изключение на:
Отговор: развитие на едематозен синдром

174. Специфичен страничен ефект, който възниква при употребата на теофилин при деца от 1-вата година от живота, е:
Отговор: мелена

175. Посочете инхалаторния глюкокортикостероид с най-ниска бионаличност:
Отговор: флутиказон пропионат

176. Посочете инхалаторно глюкокортикостероидно лекарство, което има най-нисък афинитет към глюкокортикостероидните рецептори в човешките бели дробове:
Отговор: флутиказон пропионат

177. Посочете лекарството с най-висока степен на безопасност (според индекса на безопасност:)
Отговор: преднизолон

178. Кое от глюкокортикостероидните лекарства най-много допринася за развитието на миопатия?
Отговор: триамцинолон

179. Забавянето на отделянето на натрий и вода от тялото, повишеното отделяне на калий (минералокортикоиден ефект) е по-характерно за:
Отговор: хидрокортизон

180. Минералокортикоидната активност липсва при:
Отговор: дексаметазон

181. Изберете верният отговор. Глюкокортикоиди:
Отговор: са контрацептивни хормони

182. При провеждане на импулсна терапия е по-предпочитано:
Отговор: метилпреднизолон

183. Когато се предписва за дълго време, за предпочитане е да се използват:
Отговор: преднизолон

184. Какъв H1-хистамин рецепторен блокер е противопоказан при анафилактичен шок?
Отговор: дифенхидрамин (дифенхидрамин)

185. Изберете оптималния блокер на H1-хистаминовия рецептор за лечение на алергичен ринит:
Отговор: азеластин (алергодил)

186. Посочете лекарство от групата на стабилизаторите на мембраната на мастоцитите в дозираната форма под формата на прах за инхалация:
Отговор: кромоглицинова киселина (бикромат)

187. Имуностимулатор от микробен произход включва:
Отговор: рибомунил

188. Основното показание за назначаването на рибомунил е:
Отговор: предотвратяване на рецидивиращи инфекции на горните дихателни пътища

189. Следните антибактериални лекарства проникват добре през кръвно-мозъчната бариера:
Отговор: Цефалоспорини от 3-то поколение

190. Новото поколение макролидни антибиотици има следните предимства с изключение на:
Отговор: бъбречен път на екскреция

191. Флуорохинолоните се различават от хинолоните по следните начини, с изключение на:
Отговор: бактериостатично действие

192. Проверете кои твърдения за цефалоспорините са верни:
Отговор: всичко е точно

193. Последствията от приема на антибиотици включват:
Отговор: всичко е точно

194. Посочете избраното лекарство за инфекция на пикочните пътища, причинена от Pseudomonas aeruginosa:
Отговор: цефтазидим

195. Какви лекарства са показани за лечение на хламидийна инфекция на пикочно-половия тракт:
Отговор: ровамицин

196. Посочете лекарството с най-неблагоприятни фармакокинетични характеристики:
Отговор: кетоконазол

197. Посочете антимикотично лекарство, което не се метаболизира в черния дроб:
Отговор: флуконазол

198. Посочете антимикотично лекарство (от групата на алиламините), използвано предимно за лечение на дерматомикоза:
Отговор: тербинафин

199. Посочете клиничното състояние, което е индикация за монотерапия с НСПВС:
Отговор: извънставни ревматични заболявания (миозит, тендовагинит, синовит)

200. Ацетилсалициловата киселина се характеризира с:
Отговор: когато се приема през устата, той се абсорбира главно от горната част на тънките черва

201. В сравнение с индометацин, ацетилсалициловата киселина е по-изразена:
Отговор: антитромбоцитен ефект върху тромбоцитите

202. Скоростта на екскреция на ацетилсалициловата киселина и нейните метаболити се влияе от:
Отговор: ниво на pH на урината

203. Стомашно-чревните усложнения при употребата на ацетилсалицилова киселина са свързани с:
Отговор: Всички изброени

204. Фенилбутазонът се характеризира с:
Отговор: всичко е точно

205. Когато индометацин взаимодейства с други лекарства:
Отговор: диуретичната активност на фуроземид намалява

206. Какви нежелани реакции на НСПВС се коригират от комплексния препарат Arthrotec (диклофенак натрий + мизопростол)
Отговор: НСПВС гастропатия

207. Какви характеристики на парацетамола поставят това лекарство на първо място сред аналгетиците-антипиретици?
Отговор: по-ранно начало на аналгетичен и антипиретичен ефект

208. Изберете лекарство, което селективно инхибира циклооксигеназата2:
Отговор: мелоксикам

209. Най-добър аналгетичен ефект на фентанил се наблюдава в комбинация с:
Отговор: дроперидол

210. Назовете противовъзпалително средство с продължително действие:
Отговор: пироксикам

211. При Б-о 52л се разви хипертонична криза тип 2 с пулс 62 в мин. АН 200/140 mmHg.
Отговор: Фуроземид

212. страда от захарен диабет тип 1 в продължение на 6 години, получава инсулин при 54 U / ден, което поддържа нивото на гликемия в рамките на 7,0 mmol / l. Наскоро, поради повишаване на кръвното налягане до 16090 mmHg. лекуващият лекар предписва хипотиазид в дневна доза от 75 mg в комбинация с еналаприл в доза от 5 mg.След 10 дни нивото на кръвната захар на пациента е 10,5 mmol и той се чувства по-зле. Коя е водещата причина за промени в нивата на кръвната захар?
Отговор: Комбинация от еналаприл с хипотиазид

213. развива се конвулсивна форма на хипертонична криза, състоянието е тежко, кръвното налягане е 200-120 mm Hg, сърдечната честота е 120 в минута. Кое лекарство трябва да се използва за започване на терапия?
Отговор: диазепам

214. на фона на употребата на антибиотик цефтриаксон в продължение на 10 дни се разви картина на псевдомембранозен колит. Каква е първата стъпка от алгоритъма за медицинско обслужване?
Отговор: спиране на цефтриаксон, прилагане на ванкомицин или метронидазол

215. поради обостряне на стомашна язва, кларитромицин е предписан в комплекса от терапия. Назовете основните отличителни черти на лекарството от еритромицин.
Отговор: всичко е точно

216. след операция на коремната кухина на 4-ия ден се разви левостранна пневмония на долния лоб. Резултатите от експресния анализ показват наличие на MRSA, резистентни към пеницилин и аминогликозиди щамове ентерококи. Лекарства по избор:
Отговор: Ванкомицин

217. е в интензивно отделение за Pseudomonas aeruginosa. Изберете лекарства от първа линия за лечение?
Отговор: Цефтазидим + аминогликозиди

218. 40 години без съпътстващи заболявания поради придобита в обществото пневмония на амбулаторна база, спирамицин е предписан перорално при 3 милиона IU 2 бр, на 2-ия ден от лечението са отбелязани интензивна гастралгия, гадене и еднократно повръщане. Изберете алтернативно лекарство.
Отговор: доксициклин

219. с хроничен обструктивен бронхит е открита пневмония с умерена тежест, амоксиклав е предписан перорално при 625 mg 3 rs на амбулаторна база. На 2-ия ден пациентът развива уртикария, бронхоспазъм. Назовете алтернативно лекарство за лечение на пневмония.
Отговор: моксифлоксацин през устата

220. 44-годишен ХИВ-инфектиран пациент е диагностициран с пневмоцистна пневмония. Назовете лекарството за лечение?
Отговор: ко-тримоксазол IV 20 mgkgs 4 rs за 21 дни

221. Имайте b-Noah 28l. отбелязват се ежедневни симптоми на бронхиална астма, чести екзацербации, чести нощни симптоми, диагностицирана е тежка персистираща бронхиална астма. Назовете лекарствата на основната терапия.
Отговор: инхалаторни глюкокортикоиди (повече от 1000 микрограма беклометазон дипропионат) + дългодействащи инхалаторни бета-2-агонисти

222. Бременна жена (6-7 гестационна седмица) дойде на лекар с клинични признаци на остра пневмония. Какви групи антибактериални лекарства са разрешени за употреба при бременни жени?
Отговор: цефалоспорини

223. Пациент на 57 години се лекува с АСЕ инхибитор еналаприл за умерена артериална хипертония. При пациент след 2 години прием на лекарството има липса на ефект. Кой е най-подходящият вариант за оптимизиране на терапията?
Отговор: добавяне на диуретик (хипотиазид или индапамид) към лекарството

224. получава антибактериално лекарство за инфекциозен процес. При интравенозна инфузия на лекарството се наблюдава реакция под формата на изразено зачервяване на кожата на горната половина на тялото, лицето, шията, симптомите намаляват значително с намаляване на скоростта на инфузия. Какво лекарство има такава реакция?
Отговор: Ванкомицин

225. При бременна жена има активиране на ревматичния процес. Кое лекарство от групата на антикоагулантите може да се предпише на бременна жена?
Отговор: Хепарин

226. е приет с ф-ми за повишаване на кръвното налягане до цифрите 15090 mm Hg. на фона на психо-емоционално пренапрежение, сърцебиене, тревожност, нарушение на съня. Преди година й откриха захарен диабет тип 2 и получава манинил. Назовете избраното лекарство за лечение на хипертония.
Отговор: Атенолол

227. във връзка със симптомите на ангина пекторис и ритъмни нарушения са предписани лекарства: анаприлин 200 mg и верапамил 240 mg за дълго време. Какви са възможните странични ефекти?
Отговор: Развитие на a-b блокове, брадикардия

228. Жена на 34 години приема естроген-съдържащи контрацептиви. Лекуващият лекар предписва доксициклин в доза от 200 mg за 2 седмици. Какво е вероятното очаквано взаимодействие?
Отговор: Намален ефект на контрацепцията

229. Анестезиологът предписа диазепам с профилактична цел при прилагане на кетамин. Какво състояние се предотвратява по този начин?
Отговор: Постанестезични халюцинации

230. 46 г. е приета с картина на остър деструктивен апендицит. Лекарство на избор за антибиотична профилактика?
Отговор: Цефазолин

231. Пациент се обърна към лекар с оплаквания от кашлица, температура до 39 С, болки в гърдите. Диагностицирана е дясна бронхопневмония. Предписано е лекарство за 3 дни, което има постантибиотичен ефект. Назовете избраното лекарство.
Отговор: Азитромицин

232. поради остър десен пиелонефрит е предписан цефазолин в доза от 2 g за 10 дни. Коя е най-честата грешка при избора на този антибиотик?
Отговор: Недостатъчно висока активност срещу грам-отрицателна флора

233. едновременно получава флуорохинолонов антибиотик - офлоксацин за инфекция на пикочните пътища и диклофенак натрий за ставен синдром в продължение на 14 дни. Какво е вероятното очаквано взаимодействие?
Отговор: повишен риск от възбуждане на ЦНС и гърчове

234. след хипотермия се появиха студени тръпки, повишаване на телесната температура до 38,6 ° C, кашлица със слузно-гнойни храчки, болка в лявата половина на гръдния кош.Клинично и радиологично е установена диагноза левостранна пневмония на долния лоб.на ден IM , hemodez 400 ml IV капково, отхрачваща смес 1st.l.6r на ден. На 3-ия ден имаше реакция под формата на уртикария, сърбеж по кожата. Изберете антибактериално лекарство, което да замените?
Отговор: спирамицин

235. По време на анестезията анестезиологът предписва антибактериално лекарство с цел антибиотична профилактика. Пациентът разви респираторен арест. Какво лекарство е използвано?
Отговор: Гентамицин

236. от 10 години страда от деформираща артроза на долните крайници с тежък синовит. Тя има анамнеза за лекарствена алергия към бутадион. Реопирин 5 ml интрамускулно 1 път на ден е предписан в b-noy отдел.Ден по-късно b-noy разви сърбящи еритематозни обриви по кожата на багажника.Каква е най-вероятната причина за влошаването?
Отговор: лекарствена алергична реакция

237. При сърдечни аритмии е предписан новокаинамид, а при сезонен алергичен ринит - цетиризин. Какво е вероятното очаквано взаимодействие?
Отговор: тежки форми на аритмия (тип пирует)

238. На пациент със СЛЕ е предписан метотрексат. Колко време е необходимо, за да видите стабилен терапевтичен ефект?
Отговор: бирнеше айдан кайин

239. с хипертонична криза е предписан фозиноприл. Въпреки приемането на адекватна доза от лекарството, в следващите минути и часове не се наблюдава понижение на кръвното налягане. Назовете причината.
Отговор: При кризи не се използват депо лекарства

240. за целите на планираната терапия на хипертония, апресин е предписан за дълго време. След един месец употреба на лекарството пациентът започва да изпитва сърцебиене, ангина пекторис и намаляване на ефекта от лечението. Посочете основната причина за развитите явления
Отговор: Apressin не се предписва за планирана терапия на хипертония

241. На 42 години постъпва в реанимация с тежка хипертонична криза. Натриевият нитропрусид се прилага интравенозно в продължение на 5 дни. На 6-ия ден пациентът развива картина на интоксикация под формата на неукротимо повръщане, намаляване на функцията на сърдечно-съдовата, дихателната и отделителната система. Посочете основната причина за развилото се състояние.
Отговор: Предозиране на лекарства (натрупване на тиоцианати в кръвта)

242. 54 г. страда от захарен диабет тип 1, приема удължен инсулин.Постъпила в отделението с картина на остра десностранна долнолобна пневмония, потвърдена рентгеново.Назначен е цефтриаксон, към който пациентката е имала алергична реакция Въпреки това, изследването разкрива ниско ниво на креатининов клирънс (30 ml / min) в B-noi, в резултат на което амикацин е отменен Кое лекарство трябва да продължи лечението?
Отговор: спирамицин

243. е приета в отделението с жена за болка в долната част на корема, повишаване на T. до 39,5 C. Тя се разболя преди 2 дни, на 6-ия ден след раждането. Гинекологичният преглед показа картина на остър следродилен ендометрит. Резултатите от bakposev: Staphylococcus aureus, образуващ пеницилиназа, Proteus. Определете антибиотик на първи избор
Отговор: цефепим

244. 25 л. е приета в отделението с картина на остър десен пиелонефрит.Заболява преди 3 дни след хипотермия. Предписан е цефазолин.След 2-рата инжекция на лекарството, след 10 минути, се появява понижаване на кръвното налягане, замаяност, гадене, повръщане, неволно уриниране и конвулсии. Какво усложнение се е развило при пациента?
Отговор: Анафилактична реакция

245. 28-годишна пациентка е приета в отделението с картина на остър десен мастит.Тя се разболя на 12-ия ден след раждането. , който образува пеницилиназа и кандида. Изберете антибактериално лекарство, като вземете предвид бактериалната микрофлора и фармакокинетиката
Отговор: Оксацилин + флуконазол

246. страда от хроничен холецистит.В жлъчната култура при изследването са открити Staphylococcus aureus и Escherichia coli. Тя има анамнеза за алергия към оксацилин. Лекарства по избор.
Отговор: Цефтриаксон

247. 58 години се оплакват от обща слабост, жажда, често уриниране, сърбеж по кожата и външните гениталии.При преглед: телесно тегло 56 кг при височина 168 см).Кръвна захар 12,3 mmol/l, урина 1,5%, реакцията към ацетонът е отрицателен. Какви хипогликемични лекарства са оптимални в този случай?
Отговор: Сулфонилуреи

248. 53 г., постъпва с оплаквания от сърцебиене, прекъсвания, понякога задух. Тези явления започнаха да се смущават след инфаркт на миокарда преди 2 години. Приемът на новокаинамид в продължение на 3 месеца донесе значително облекчение. През последните години обаче здравословното състояние се влоши. По-нататъшно управление на пациента.
Отговор: пълен преглед и избор на лекарството

249. Пациент на 33 години получава комбинация от лекарства за постоперативен гноен перитонит: цефтриаксон + амикацин + метронидазол. Тя има анамнеза за жлъчнокаменна болест. Назовете лекарството, което не трябва да се предписва на пациента.
Отговор: Цефтриаксон

250. На 45-годишна жена с кандидозна пневмония е предписан флуконазол IV за 3 дни, след това перорално. На 4-ия ден от лечението се наблюдава реакция под формата на силно главоболие, гадене. Беше решено да се замени лекарството с кетоконазол. Оценявайте адекватността на тактиката.
Отговор: Кетоконазол не е адекватен заместител поради неблагоприятни фармакокинетични характеристики

251. 42 години, е приет с оплаквания от силно сърцебиене, повишаване на кръвното налягане до 240140 mm Hg. Изследването разкрива значително повишаване на нивото на катехоламините в кръвта. Назовете лекарството по избор за спиране на кризата.
Отговор: фентоламин

252. 50 години, постъпва с картина на остър левостранен пиелонефрит. Предписани са цефазолин + гентамицин в средни терапевтични дози. По време на изследването креатининовият клирънс на пациента е 50 ml min. Какви са вероятните последици от терапията?
Отговор: Риск от нефротоксичност

253. На 48 години с пароксизмална суправентрикуларна и камерна тахикардия е предписан кордарон IV на първия ден, след това вътре. При прегледа се установи дисфункция на щитовидната жлеза, блокада 2-3 степен. Допълнителни тактики.
Отговор: Анулиране на лекарството, назначаване на новокаинамид

254. От 50 години е в кардиоинтензивното отделение за остър миокарден инфаркт, получава комплексна терапия. Какви са основните параметри за наблюдение при предписване на директни антикоагуланти?
Отговор: APTT, време на съсирване на кръвта, урина за еритроцити

255. Пациент е в отделението след операция на апендицит. Лекуващият лекар предписва гентамицин 80 mg 3 пъти на ден. Пациентът имаше остър гломерулонефрит преди 2 години, в момента креатининовият клирънс е 50 ml min. Каква корекция е необходима?
Отговор: замяна с лекарство, което няма нефротоксичност

256. 56 г., получава дигоксин по 0,25 g през последната година. Понастоящем започва да се отбелязва повишаване на стойностите на кръвното налягане до 180110 mm Hg. Лекуващият лекар предписва лизиноприл в доза от 10 mg. След 3 месеца пациентът е диагностициран с дигиталисова интоксикация. Допълнителни тактики.
Отговор: предписване на друго антихипертензивно лекарство

257. 53-годишен е диагностициран с кандидозен и аспергилозен менингит. Лекарства по избор.
Отговор: Амфотерицин Б

258. 58-годишен пациент, получаващ дълго време глюкокортикоиди, е включен в комплексната терапия със синтетичен широкоспектърен антибиотик поради инфекция на репродуктивната сфера. На 14-ия ден от съвместната употреба се наблюдава тежко усложнение под формата на разкъсване на ахилесовото сухожилие. Посочете антибиотика, който в комбинация с кортикостероиди е причинил това усложнение.
Отговор: Левофлоксацин

259. Б.43 г. е приета в отделението с картина на остра хламидийна пневмония. Назовете лекарствата по избор.
Отговор: Ровамицин

260. Пациент на 24 години е диагностициран с неусложнена форма на гонорея. Назовете избраното лекарство.
Отговор: Цефтриаксон

261. Изберете правилното твърдение: а) бионаличността е количеството лекарства, влизащи в системното кръвообращение, изразено като процент от приетата доза, б) бионаличността се определя от степента на адсорбция на лекарството в стомашно-чревния тракт и изразена
ефектът от първото преминаване през черния дроб в) бионаличността се определя по формулата: F = AUC (в / m или перорално) / AUC (в / в) г) бионаличността на лекарствата при интрамускулно приложение се определя от степента на неговото усвояване и биотрансформация в организма.
Отговор: a B C

262. страда от неатопична бронхиална астма, придружена от профузна бронхорея Пулс 62 мин. АН 140/80 mm Hg Кои лекарства са за предпочитане?
Отговор: Атровент

263. Механизми на абсорбция на лекарства в червата:
а) пасивна дифузия, б) филтрация, в) активен транспорт, г) улеснен транспорт, д) пиноцитоза:
Отговор: а, г

264. е приет за киселини, болка в епигастралната област на празен стомах, облекчена от натриев бикарбонат FEGDS разкрива язва (0,5 cm в диаметър) в 12 p.
zuyuschaya функция със средна интензивност с ниски алкални резерви, холинергичен тип рецепция. Диагноза: пептична язва 12 бр в остър стадий. Изберете най-ефективното и безопасно лекарство и определете неговия режим на дозиране:
Отговор: Пирензепин преди хранене 0,05 g 3 пъти на ден в продължение на 2 дни, след това 0,05 g 2 пъти на ден

265. открита дискинезия на хипертоничен тип на жлъчния мехур. Изберете най-добрия вариант за лечение.
Отговор: No-shpa 1-2 таблетки 3 пъти на ден, отвара от безсмъртниче 1/2 чаша 30 минути преди хранене

266. страда от хроничен холецистопанкреатит в продължение на 5 години.През последната седмица след нарушение на диетата се наблюдава засилване на болката в дясната гръдна клетка, гадене, горчивина в устата.Изберете най-ефективните холеретични средства, които едновременно имат антимикробна активност:
а) алохол, б) холензим, в) никодин, г) отвара от вратига,
д) Ксилитол
Отговор: а, в

267. с намерение за самоубийство изпила 20 таблетки феназепам.2 часа след приема на лекарството била откарана в болница. B-I е в съзнание, но рязко потиснат.Извършена е стомашна промивка.Изберете най-оптималните лаксативи: а) глауберова сол, б) магнезиев сулфат, в) екстракт от кора на зърнастец, г) бисакодил,
д) рициново масло, е) морски водорасли, ж) вазелиново масло
Отговор: а,б,г

268. Мъж на 46 години е приет в кардиоинтензивно отделение с остър трансмурален инфаркт на миокарда, настъпил преди около 5 часа.Назначения: анаприлин 20 mg 4 пъти дневно перорално, хепарин интравенозно по 10 000 единици на всеки 4 часа.В същото време време, беше възможно да се постигне увеличаване на времето за съсирване на кръвта до 18-23 минути. На 4-ия ден пациентът има микрохематурия (22 еритроцита в зрителното поле). Каква е вашата тактика?
Отговор: Намалете дозата на хепарина, докато времето за съсирване стане поне 10-12 минути

КАТЕГОРИИ

ПОПУЛЯРНИ СТАТИИ

2023 "kingad.ru" - ултразвуково изследване на човешки органи